Quick viewing(Text Mode)

CLAT 2017 Question Paper Pdf Download

CLAT 2017 Question Paper Pdf Download

www.gradeup.co

1 www.gradeup.co

Direction (1 – 10) : Fill in the blank by 8.The passengers were very happy _____ choosing the most appropriate option. the friendly and warm treatment. 1. A. To We shall fail ____ we are industrious. B. From A. Whether C. About B. Unless D. By C. Until D. Though 9.If they want to succeed, ____ they have to work very hard. 2.She stood ___ Amit but could not utter A. Should a single word for quite some time. B. Ought A. Before C. Will B. For D. Must C. About D. To 10.Sunita decided to set ____ some time every day for prayers. 3.Kanak is endowed ____ many great A. Up qualities. B. In A. By C. Aside B. With D. On C. In D. Of Direction (11 – 15) : Read the given passage carefully and choose the most 4.The minister flew ____ the flooded appropriate option to the questions given areas in a helicopter. below. A. Along The World Trade Organisation (WTO) was B. Over created in the early 1990s as a C. In component of the Uruguay Round D. About negotiation. However, it could have been negotiated as part of the Tokyo Round of 5.You have played a great role, for ____ the 1970s, since negotiation was an your help I possibly would have landed attempt at a 'constitutional reform' of the myself into a problem. General Agreement on Tariffs and Trade A. Without (GATT). Or it could have been put off to B. Although the future, as the US government C. Despite wanted. What factors led to the creation D. After of the WTO in the early 1990s? One factor was the pattern of multilateral bargaining 6.The doctor advised him to go ____ that developed late in the Uruguay several medical tests. Round. Like all complex international A. About agreements, the WTO was a product of a B. Under series of trade-offs between principal C. Through actors and groups. For the United States, D. Into which did not want a new organization, the disputed settlement part of the WTO 7.Would anybody ____ a mother have package achieved its longstanding goal of risked her life for the baby? a more effective and more legal dispute A. But settlement system. For the Europeans, B. Rather who by the 1990s had come to view GATT C. Than dispute settlement less in political terms D. However add more as a regime of legal obligations, the WTO package was acceptable as a means to discipline the resort to

2 www.gradeup.co

unilateral measures by the United States. settlement procedures and institutional Countries like Canada and other middle provisions. Concern for these values is and smaller trading partners were inherent in any rule-based system of co- attracted by the expansion of a rule- operation, since without these value rules based system and by the symbolic would be meaningless in the first place, valueof a trade organization, both of therefore, create their own incentive for which inherently support the weak fulfilment. The moment of legal against the strong. The developing development has occurred in other countries were attracted due to the institutions besides the GATT, most provisions banning unilateral measures. notably in the European Union (EU). Over Finally, and perhaps most important, the past two decades the European Court many countries at the Uruguay Round of Justice (ECJ) has consistently rendered came to put a higher priority on the decisions that have expanded export gains than on the import losses incrementally the EU's internal market, in that the negotiation would produce, and which the doctrine of 'mutual recognition' they came to associate the WTO and a handed down in Cassis de Dijon case in rule-based system with those gains. This 1979 was a key turning point. The court reasoning - replicated in many countries is now widely recognized as a major - was contained in U. S. Ambassador player in European integration, even Kantor's defence of the WTO, and it though arguably such a strong role was announced to a recognition that not originally envisaged in the Treaty of international trade and its benefits cannot Rome, which initiated the current be enjoyed unless trading nations accept European Union. One means the Court the discipline of a negotiated rule-based used to expand integration was the environment. A second factor in the 'teleological method of interpretation', creation of the WTO was pressure from whereby the actions of member states lawyers and the legal process. The were evaluated against 'the dispute settlement system of the WTO accomplishment of the most elementary was seen as a victory of legalists but the goals set forth in the Preamble to the matter went deeper than that. The GATT, (Rome) treaty. The teleological method and the WTO, are contract organizations represents an effort to keep current based on rules, and it is inevitable that an policies consistent with slated goals, and organization creating a further rule will in it is analogous to the effort in GATT to turn be influenced by legal process. keep contracting party trade practices Robert Hudee has defined the consistent with slated rules. In both cases "momentum of legal development". But legal concerns and procedures are an what is it precisely? Legal development independent force for further co- can be defined as the promotion of operation. technical and legal values of consistency, In the large part the WTO was an exercise clarity (or certainty) and effectiveness. in consolidation. In the context of a trade These are the values for the ones who are negotiation that created a near- responsible for administering any legal revolutionary expansion of international system will seek to maximize. As it played trade rules, the formation of the WTO was out in the WTO, consistency meant a deeply conservative act needed to integrating under one roof the whole lot ensure that the benefits of the new rules of separate agreements signed under would not be lost. The WTO was all about GATT auspices; clarity meant removing institutional structure and dispute ambiguities about the powers of settlement: these are the concerns of contracting parties to make certain conservatives and not revolutionaries, decisions or to undertake waivers; and that is why lawyers and legalists took the effectiveness meant eliminating lead on these issues. The WTO codified exceptions arising out of grandfather- the GATT institutional practice that had rights and resolving defects in dispute developed by custom over three decades,

3 www.gradeup.co

and it incorporated a new dispute 14.According to the passage, WTO settlement system that was necessary to promoted the technical legal values partly keep both old and new rales from through. becoming a sham. Both the international A. Integrating under one roof the structure and the dispute settlement agreements signed under GATT. system were necessary to preserve and B. Rules that create their own incentive enhance the integrity of the multilateral for fulfilment. trade regime that had been built C. Ambiguities about the powers of incrementally from the 1940s to the contracting parties to make certain 1990s. decisions. D. Grandfather-rights exceptions and 11. defects in dispute settlement procedures. In the statement ‘... it amounted to a recognition that international trade and 15.The most likely reason for the its benefits cannot be enjoyed unless acceptance of the WTO package by trading nations accept the discipline of a nations was that: negotiated rule-based environment', it A. It has the means to prevent the US refers to: from taking unilateral measures. A. The export gains many countries came B. Its rule-based system leads to export to associate with a rule-based system. gains. B. The higher priority on export gains C. It settles disputes more legally and placed by many countries at the Uruguay more effectively. Round. D. They recognized the need for a rule- C. The provision of a rule-based system based environment to protect the by the WTO benefits of increased trade. D. Ambassador Kantor's defence of the WTO Direction (16 – 20) : In each of the following sentences, some part of the 12.What would be the closest reason why sentence or the entire sentence is WTO was not formed in 1970s? underlined. Beneath each sentence, you A. The US government did not like it. will find four ways of phrasing the B. Important players did not find it in underlined part. Choose the most their best interest to do so. appropriate option given in each of the C. Lawyers did not work for the dispute sentences given below that is the best settlement system. version than the underlined part of the D. The Tokyo Round negotiations was an sentence. attempt at constitutional reform. 16. Two valence states of uranium, one with 13.In the method of interpretation of the a deficit of four electrons and the other European Court of Justice: one with a deficit of six occurs in nature A. Actions against member states needed and contributes to the diversity of to be evaluated against the said uranium’s behaviour. community goals. A. The other one a deficit of six, occur in B. Enunciation of the most elementary Nature and contribute community goals needed to be B. The other with a deficit of six, occurs emphasized. in Nature and contributes C. Current policies need to be consistent C. The other with a deficit of six, occur in with stated goals. Nature and contribute D. Contracting party trade practices need D. One with six occurs in Nature and to be consistent with stated rules. contributes

4 www.gradeup.co

17.Initiative and referendum, is a C. With everyone attending not procedure that allows voters to propose knowledgeable. and pass laws as well as to repeal them. D. For not everyone attending knew. A. Allows voters to propose, pass and to repeal laws Direction (21 – 24) : Choose the correct B. Will allow laws on be proposed, spellings in questions given below. passed, as well as repealed by voters 21. C. Allows voters to propose to pass, and Choose the correct spellings in options repeal laws given below. D. Will allow voter to propose, pass, as A. Accommedation well as to repeal laws. B. Accommodation C. Accomadation 18.Plausible though it sounds, the D. Accomedation weakness of the hypothesis is that it does not incorporate all relevant facts. 22.Choose the correct spellings in options A. The weakness of the hypothesis which given below. sounds plausible. A. Ghallows B. Though the hypothesis sounds B. Gallows plausible, its weakness C. Ghellows C. Even though it sounds plausible, the D. Gellows weakness of the hypothesis D. Though plausible, the hypothesis’ 23.Choose the correct spellings in options weakness given below. A. Renumeration 19.Many of them chiselled from solid rock B. Remuneration centuries ago the mountainous regions C. Remunaration are dotted with hundreds of monasteries: D. Renumaration A. The mountainous regions are dotted with hundreds of monasteries, many of 24.Choose the correct spellings in options which are chiselled from solid rock, given below. centuries ago. A. Blashphemy B. The mountainous regions are dotted B. Bleshphemy with hundreds of monasteries, many of C. Blasphamy them chiselled from solid rock, centuries D. Blasphemy ago. C. Hundreds of monasteries, many of Direction (25 – 28) : Choose the correct them chiselled from solid rock centuries spellings in questions given below. ago, are dotting the mountainous 25. regions. Choose the correct spellings in options D. Chiselled from solid rock centuries given below. ago, the mountainous regions are dotted A. Hyphothecation with many hundreds of monasteries. B. Hypathecation C. Hypothecation 20.During her lecture, the speaker tried D. Hypthacation to clarify directional terms, for not everyone in attendance was 26.Choose the correct spellings in options knowledgeable that winds are designed given below. by the direction from which they come. A. Gratuitous A. With everyone in attendance not B. Gratitious knowing. C. Gratetious B. For everyone in attendance did not D. Gratuitus know.

5 www.gradeup.co

27.Choose the correct spellings in options 33.The hope (A) to go through (B) the given below. book today I have (C) almost read (D) A. Interrogation half of it. B. Interogetion A. A C. Interogation B. B D. Interagation C. D D. C 28.Choose the correct spellings in options given below. 34.Our boss always (A) asks us to pay (B) A. Annulement full (C) attention to the work at hand (D). B. Anulment A. B C. Annulment B. A D. Annulmant C. D D. C Direction (29 – 30) : Choose the correct spellings in questions given below. 35.On listening (A) to the confession of 29. love (B) she blushed until (C) she was Choose the correct spellings in options purple (D). given below. A. A A. Abhayence B. C B. Abheyance C. D C. Abeyance D. B D. Abeyence 36.After being finished (A) the (B) last 30.Choose the correct spellings in options chapter of the book return it (C) to me given below. (D). A. Moratarium A. C B. Moretorium B. A C. Maratorium C. D D. Moratorium D. B

Direction (31 – 40) : In each of the 37.Since (A) I have forgotten (B) all following sentences four words or phrases equations I will have (C) to start from the are underlined. If there is any mistake scratch (D). with regard to grammar or usage, it is in A. B the underlined part only. Identify the B. A incorrect part. C. C 31. D. D Evidently (A) our product is the most(B) unique in(C) the market(D). 38.He gave me (A) a ticket so that (B) I A. C may visit (C) the (D) book fair. B. D A. C C. B B. A D. A C. B D. D 32.The most (A) difficult job is to bend (B) and then lifting (C) the weight (D). 39.Five gallons of (A) petrol are (B) not A. D enough (C) to cover the distance (D). B. B A. A C. A B. B D. C C. D D. C

6 www.gradeup.co

40.The officer asked that (A) the report 47.Which of the following individuals was (B) be submitted (C) immediately (D). called a 'deceptive actor' by China's A. D foreign ministry during March 2017? B. C A. Narendra Modi C. A B. Donald Trump D. B C. Dalai Lama D. Sirisena 41.Prithvi Defence Vehicle is: A. The name of a newly developed tank 48.Who called the immigration the B. The name of a single seated aircraft 'Trojan horse of Terrorism'? developed by HAL A. Donald Trump, President of the US C. The name of 's Nuclear B. Angela Merkel, Chancellor of Germany Interceptor Missile C. Viktor Orban, Prime Minister of D. The name of an amphibious Naval vessel D. Theresa May, Prime Minister of U.K

42.Highest number of open prisons in 49.How many billionaires India has lost India as on 2015 are in: since demonetization on November 8, A. Rajasthan 2016? B. Maharashtra A. Two C. Tamil Nadu B. Eight D. Kerala C. Eleven D. Eighteen 43.The first Commercial Court and Commercial Disputes Resolution Centre 50.Donald Trump is President…………of the was inaugurated at United States. A. , Maharashtra A. 43rd B. Ahmedabad, Gujarat B. 44th C. Raipur, Chhattisgarh C. 45th D. Visakhapatnam, Andhra Pradesh D. 46th

44.The world's first artificial intelligence 51.'Scorpion kick' is a phrase used in lawyer, a robot, is named as: A. Kung Fu A. Ross B. Kabaddi B. Boss C. Kick Boxing C. Watson D. Kalari Fight D. IBM-LaBrain 52.India's third largest trading partner 45.Who has been selected for 2016 BC during 2016 is Roy Award? A. Saudi Arabia A. Dr. Jagat Ram B. Kuwait B. Dr. P. Raghu Ram C. U.A.E C. Dr. N. Bhaskaran D. Dubai D. Dr. J. Rajendra 53.The Union Cabinet has recently 46.According to the Survey Report approved the setting up of a Permanent released by Transparency International Tribunal for resolving: during March 2017 on India, the most A. Inter-state water disputes corrupt are B. Election disputes A. Government officials C. Inter-state boundary disputes B. Business executives D. Complaints from three services C. Police D. Local Councilors

7 www.gradeup.co

54.Among the following M.L.As, who was D. Small ships disqualified by the Governor during January 2017 under Article 192 of the 61.Which court has stayed the execution Constitution for undertaking government of death sentence of Kulbhushan Jadhav contracts? in May 2017? A. Uma Shankar Singh of Bihar A. Supreme Court of India B. Jayalalithaa of Tamil Nadu B. Supreme Court of Pakistan C. Srinivas Prasad of C. International Court of Justice D. PC. George of Kerala D. International Criminal Court

55.Which shoe company in the United 62.The Japanese Prime Minister who States of America has won an IPR dispute offered 'sincere and everlasting against China recently for using their condolences' to the people of the United logo? States for killing more than 2,400 soldiers A. Reebok in the attack on Pearl harbour was: B. Nike A. Hayato Ikeda C. New Balance B. Shinzo Abe D. Adidas C. Kakuei Tanaka D. Juniciro Koizumi 56.The World Bank had cut India's GDP growth for 2016 - 2017 to: 63.The Hubble telescope of NASA is A. 7.6% located in ___. B. 8% A. NASA headquarters C. 7% B. Space D. 8.6% C. D. Canada 57.Among the following who was crowned as 'Miss Supernational' during 2016? 64.The first country in the world to have A. begun shutting down the entire B. Aishwarya Rai Frequency Modulation (FM) radio network C. Shilpa Shetty to be replaced by Digital Audio D. Alia Bhatt Broadcasting is: A. United States of America 58.In terms of steel production in the B. Switzerland world during 2015-2016, India stood at: C. China A. Second D. B. Third C. Fourth 65.Name the President elect of D. Fifth who is likely to take the oath on 14 May 2017. 59.Volvo has launched the world's largest A. Francois Holland bus that can carry up to: B. Marine Le Pen A. 150 passengers C. Francois Fillon B. 260 passengers D. Emmanuel Macron C. 300 passengers D. 320 passengers 66.Till the end of 2016, the total number of UNESCO's World Heritage Sites in India 60. is threatening to drag India to is: W.T.O on issues relating to the export of A. 42 its: B. 18 A. Steel C. 35 B. Tea D. 21 C. Electronic goods

8 www.gradeup.co

67.Among the following, who has won the 74.Juno is the name of a: maximum number of titles? A. Solar powered space craft A. P. V. Sindhu B. Hydrogen fuelled space craft B. Gopichand C. Atomic powered submarine C. Saina Nehwal D. Hydrogen fuelled submarine D. Srikant 75.The first country to have announced 68.Among the following professors of euthanasia of a child is: Indian origin who has received A. Norway Knighthood for the work as a co-inventor B. Denmark of next generation DNA Sequencing called C. Finland Solexa Sequencing is? D. Belgium A. Shankar Balasubramanian B. P. C. Mahalanobis 76.Which of the following country C. Satyendra Nath Bose enacted a law during August 2016 D. Mehnad Saha providing for the right to register the marriages of Hindus? 69.'World's longest-all women Non-stop A. Afghanistan flight' from New Delhi to San Francisco B. Pakistan covering 14,500 kms was operated by: C. Saudi Arabia A. Jet Airways D. Iran B. Air India C. United Airlines 77.The 2016 Nobel Peace Prize was won D. American Airlines by the President of: A. United States of America 70.India's voting rights at the B. International Monetary Fund increased C. Columbia from 2.3 % to D. A. 2.4% B. 2.5% 78.Immediately before Antonio Guterres C. 2.6% was appointed the U.N Secretary General D. 2.8% in October 2016, he was: A. The Prime Minister of Portuguese 71.The first elected civilian President in B. United Nations High Commissioner for is: Refugees A. Htin Kyaw C. Commissioner General of UNRWA B. Aung San Suu Kyi D. United Nations High Commissioner for C. Aung San Human Rights D. Khin Ayi 79.With the development of Terahertz 72.World's longest rail tunnel is about: (THs) transmitter, it is expected to be A. 23 kms. faster than 5G mobile networks by: B. 32 kms. A. Two times C. 47 kms. B. Five times D. 57 kms. C. Ten times D. Four times 73.Solar Impulse-2 is: A. Impact of climate change for overall 80.The bowler who has claimed the increase of 2° C a year fastest 250 wickets in Cricket test B. Solar powered ship matches is: C. Solar powered airplane that completed A. Anil Kumble the first around the world B. Dennis Lillee D. Hybrid airplane C. Rangana Herath

9 www.gradeup.co

D. Ravichandran Ashwin D. Bhutan

81.The Happiness Index Department or a 88.As on 31st January 2016, the highest Wing has been established in the states number of law colleges were present in: of: A. Madhya Pradesh A. Andhra Pradesh and Madhya Pradesh B. Uttar Pradesh B. Madhya Pradesh and Tamil Nadu C. Maharashtra C. Andhra Pradesh and Sikkim D. Andhra Pradesh D. Madhya Pradesh and Goa 89.The first statue of a woman in 82.The top destination for domestic Parliament Square in England is that of: tourists in India for the past three A. Margaret Thatcher consecutive years has been: B. Mother Theresa A. Kerala C. Queen Elizabeth - II B. Orissa D. Millicent Fawcett C. Tamil Nadu D. Rajasthan 90.India emerged as …………. largest holder of the U.S Government Securities 83.COIN, a software programme at the end of 2016. developed by J. P. Morgan supports: A. Twenty fourth A. Bitcoin B. Twelfth B. Robotic surgery C. Twenty eighth C. Financial accounting D. Fifteenth D. Interpreting commercial documents 91.Principle: Acceptance of a proposal 84.NASA rediscovered India's lunar must be absolute and unqualified. spacecraft that was lost in the space Facts: 'A' made a proposal to sell his during the past eight years known as: motorcycle to 'B' for rupees 25,000/-. 'B' A. Chandrayan - I agreed to buy it for rupees 24,000/-. 'A' B. Chandrayan - II sold his motorcycle to 'C for 26,000/- the C. Mangalayaan - I next day. 'B' sues 'A' for damages. D. Mangalayaan – II A. 'B' will not get any damages from 'A' B. 'B' will get damages from 'A' 85.Which country 3D - Printed a home of C. 'B' will get the difference of rupees 37 sq. mts? 1,000/- only A. Spain D. 'B' can proceed against 'C B. C. 92.Principle: Contract is a written or D. Taiwan spoken agreement, with specific terms between two or more persons or entities 86.Which country offered asylum seekers in which there is a promise to do 1,200 Euros to leave by withdrawing their something in return for a valuable benefit application for protection? known as consideration. Such an A. France agreement is intended to be enforceable B. by law. A unilateral contract is one in C. Germany which there is a promise to pay or give D. Denmark other consideration in return for actual performance. 87.During 2017, which SAARC country Facts: A Toilet Soap Manufacturing has notified the Right to Information Act? Company in India in order to promote the A. sale of their product, published an B. Sri Lanka advertisement in all the Newspapers on C. Afghanistan January 1, 2017 that the Company has

10 www.gradeup.co

kept a model ignition key of an Audi A3 94.Principle: It is a case of fraud where Car. The advertisement also stated that a party to a contract knows or believes a whoever gets the said key before fact to be true, but conceals it actively December 31, 2017 from a soap bar will from the other party with a view to induce be gifted with the Audi A3 Car. Mr. Martin, that person to enter into the contract. a foreigner who came to India as a Tourist Facts: While taking a life insurance who was staying in a Hotel found a Key policy, in reply to questions by the similar to same Car Ignition Key. Mr. insurance company during the inquiry Martin brought this matter to the notice into his proposal, Zameer deliberately of the Hotel Manager. The Manager concealed the fact of his medical informed Mr. Martin about the Company's treatment for a serious ailment, which he advertisement on January 1, 2017. Mr. had undergone only a few weeks ago. Martin wants to claim the Car. Will he A. The concealment of fact by Zameer succeed? amounted to fraud. 1) No. The Soap Company has not B. The act of Zameer amounted to entered into a contract with Mr. Martin as innocent misrepresentation. he was not in India on January C. The act of Zameer did not amount to A. 2017 when the advertisement was fraud, as disclosing the fact would have published. resulted in exposure of his privacy. B. Mr. Martin obtained the Key before the D. The act of Zameer did not amount to stipulated date from the Soap Bar. So he any misrepresentation. is covered by the offer of the Soap Company and can claim the car. 95.Principle: Every agreement, of which C. The Hotel Manager who could legally the object or consideration is opposed to claim the Car as he was the one actually public policy, is void. An agreement which purchased the soap for the use in the has the tendency to injure public interest Hotel. or public welfare is one against public D. No. Actual intention of the Company policy. What constitutes an injury to was to promote the sale of the Soap. public interest or public welfare would depend upon the times and the 93.Principle: Where one of the parties to circumstances. a contract was in a position to dominate Facts: 'A' promises to obtain for 'B' an the decision of the other party, the employment in the public service, and 'B' contract is enforceable only at the option promises to pay rupees 5,00,000/-to 'A'. of the party who was in a position to A. The agreement is valid, as it is with dominate the decision of the other party. consideration for public service. Facts: A doctor asked his patient to make B. The agreement is valid, as it is a a payment of rupees Ten Lakh for contract between two parties with their treatment of his fever. The patient paid free consent. an amount of rupees Five Lakh and C. The agreement is void, as the object promised to pay the remaining amount and consideration for it is opposed to after the treatment. After treatment the public policy. patient recovered from fever. The doctor D. The agreement is void because rupees demanded the remaining amount from 5,00,000/- is excessive. the patient. The patient refused to pay. A. The contract is enforceable against the 96.Principle: Assault is causing bodily doctor. injury to another person by use of B. The contract is enforceable against the physical force. patient by the doctor. Facts: Rustum while entering into C. The contract is not enforceable as compartment of a train raised his fist in doctor was in dominating position. anger towards a person Sheetal, just in D. The contract is not enforceable without front of him in the row, to get way to the consent of the patient.

11 www.gradeup.co

enter into the train first, but did not hit Facts: 'P’ submitted a written consent to him. Rustum has: a surgeon 'S' for undergoing a surgical A. committed an assault on Sheetal operation for removal of appendicitis. The B. not committed an assault on Sheetal surgeon while doing surgery also C. insulted Sheetal removed the gall bladder of ‘A’: D. Rightly showed his anger A. 'P' Can claim damages from 'S' B. 'P' cannot claim damages from 'S' 97.Principle: Ownership in property C. ‘P’ is not bound to pay expenses of the consists of right to possess, right to use, surgery right to alienate and right to exclude D. ‘P’ is required to pay expenses for others. Sale is complete when property surgery for Appendicitis but not for Gall gets transferred from the seller to the Bladder buyer on sale. Facts: 'A' sold his car to 'B'. After this, 'B' 100.Principle: An agreement, the terms requested 'A' to keep the car in his care of which are not certain, or capable of on behalf 'B' for one month. 'A' agreed. being made certain, is void. A. Sale of car is complete. Facts: Sunder agreed to take Bhola's B. Sale of car is not complete penthouse on rent for three years at the C. Sale will be completed when 'B' will rate of rupees 12, 00, 000/- per annum take the delivery of the car. provided the house was put to thorough D. Sale will be automatically completed repairs and the living rooms were after the expiry of one month decorated according to contemporary style. 98.Principle: When a person who has A. There is no valid contract because it made a promise to another person to do has vague and uncertain terms, as the something does not fulfill his promise, the term 'present style' may mean one thing other person becomes entitled to receive, to Sunder and another to Bhola. from the person who did not fulfill his B. There is a valid contract because there promise, compensation in the form of is an offer from Sunder and acceptance money. from Bhola Facts: 'X' made a promise to 'Y' to repair C. It is voidable contract at the option of his car engine. 'Y made the payment for Bhola. repair. After the repair, 'Y went for a drive D. There is a valid contract because all in the same car. While driving the car, 'Y the terms of contract are certain and not met with an accident due to bursting of a vague as the rent is fixed by both of them tyre. and the term 'present style' only can be A. 'X' will be entitled to receive interpreted to mean the latest style. compensation from 'Y’ in the form of money. 101.Principle: Whoever takes away with B. 'Y will be entitled to receive him any minor less than sixteen years of compensation from 'X' in the form of age if a male, or less than eighteen years money. of age if a female, out of the custody of C. 'X' will not be entitled to receive parents of such minor without the compensation. consent of such parents, is said to commit D. 'Y’ will not be entitled to receive no offence. compensation from 'X'. Facts: 'A', a man, took away a girl below sixteen years to Mumbai without 99.Principle: When a person consented informing the parents of the girl. to an act to be done by another, he A. 'A' committed an offence against the cannot claim any damages resulting from girl. doing that act, provided the act done is B. 'A' committed no offence against the the same for which consent is given. parents of the girl.

12 www.gradeup.co

C. 'A' committed an offence against the Facts: A' writes a letter to 'E' in which he girl as well as her parents uses abusive language against 'B' and D. 'A' committed no offence against the also states that 'B' is a dishonest person. girl as well as her parents. 'A' put the letter in a sealed envelope and delivered it to 'B'. 102.Principle: When a person interferes A. 'A' has committed defamation with peaceful possession of another B. 'A' has not committed defamation person without the permission of the C. A' has not committed moral wrong person in possession of those premises, D. 'A' has committed a moral wrong commits trespass to land. Facts: 'T1 just walked over the land of P1 106.Principle: If a party to a contract to reach his house as it was a short cut. agrees to it under undue influence of any P1 had displayed a notice that it is not a other party then the party under the thoroughfare. P1 did not cause any undue influence may refuse to perform in damage to the land. accordance with the agreement. A. 'T’ has violated privacy of 'P' Facts: A, a rich youngster became a B. ‘T’ has created nuisance for ‘P’ member of a religious group and soon he C. ‘T’ has committed trespass to land was appointed by P the head of the group D. ‘T’ has not committed any trespass on as his personal secretary. As per the rules the land of ‘P’. of the group, all officials and staff of the group were supposed to stay in the 103.Principle: Penal laws provide that group's official premises itself. Some days whoever voluntarily has carnal later, A was asked by P to execute a Gift intercourse against the order of nature deed in favour of P, in which it was with any man or woman, shall be mentioned that all immovable properties punished for rape. in his name are being gifted to P. A was Facts: A Police Officer found a man unwilling to execute the deed, but he was engaged in carnal intercourse with an forcefully restrained by P and his body animal. The Police Officer arrested the guards in P's office and made A sign the man and produced him before the Court. gift deed. Soon after this A left the group A. Court will punish the police officer. and refused to hand over the property as B. Court will not punish the man for rape. agreed to in the gift deed. Is A's action C. Court will punish the man for rape. valid? D. Court will not punish the police officer. A. As the gift deed was executed by A, he cannot refuse. 104.Principle: Nothing is an offence B. As Gift is also a contract, the consent which is done in the exercise of the right of A was not obtained by P while of private defence. executing the deed. Facts: 'A', under the influence of C. A executed the deed, under madness, attempts to kill 'B’. 'B' to save compulsion and undue influence, and was his life kills 'A'. right in withdrawing from the contract. A. 'A' has committed the offence of D. It is illegal for religious groups acquire attempt to murder. property from its members. B. 'A' has not committed an offence because he was mad. 107.Principle: The concept of natural C. 'B' has committed an offence. justice is against bias and for the right to D. 'B' has not committed any offence. a fair hearing. While the term natural justice is often retained as a general 105.Principle: When a person makes concept, and it has largely been replaced such a statement which lowers other and extended by the general 'duty to act person's reputation in the estimation of fairly’. other persons, is liable for committing Fact: 'X', a male employee of a company defamation. was dismissed by the employer just on

13 www.gradeup.co

the basis of a complaint by 'Y', a female D. A is liable for the negligence as he employee of the company that 'X' was failed to take proper care during the trying to be too friendly with her and surgery. often requested her to accompany him to the canteen. 110.Principle: A contract would be Is the dismissal of 'X' valid? invalid and unlawful, if the contract is for A. Yes, because men are not supposed to an immoral or illegal purpose. behave improperly with women and Facts: P, was a young and helpless hence there is no violation of any widow, living on the pavement. R, a principles of law neighbour gave her a house, registered in B. No, because in the modern times this her name, on the condition that she type of behaviour is common should allow R to keep his smuggled C. Yes, moral law is antique and goods and drugs in her house. After the therefore, not applicable in modern registration was done, according to the times, therefore the termination is valid condition in the contract, R's agents went and no violations of the principles of to keep some packets in her house, she natural justice occurred refused. R told her the condition under D. No, because the employer did not give which the house was given to her. She a chance to 'X' to explain his side, thereby still refused. Is P justified in her action? violated the principles of natural justice. A. P is not justified as she did not have the right to deny R's request. 108.Principle: A person is said to do a B. As R was making the contract for illegal thing fraudulently, if he does that thing activities, P's stand is valid in law. with intent to defraud, but not otherwise. C. R can take back the house by Facts: ‘A’ occasionally hands over his ATM cancelling the transfer deed. card to 'B’ to withdraw money for ‘A’. On D. P is right as she did not like smuggled one occasion 'B' without the knowledge of goods to be kept in her house. ‘A’, uses A's ATM card to find out the balance in A's account, but does not 111.Principle: Under the Employees withdraw any money. Compensation Act, 1923, an employer is A. 'B' has committed the act fraudulently liable to pay compensation to his B. 'B' has not committed the act workmen for injuries sustained by them fraudulently by an accident arising out of and in the C. 'B' has committed misappropriation course of employment. D. 'B' has committed breach of faith Facts: M, the Manager of SRK Industries asked his secretary S to submit a report 109.Principle: Negligence is actionable at the Government Labour Office. 'S' in law. In simple terms, negligence is the submitted the report as directed. On his failure to take proper care over way back S met one of his class mates. something. He then decided to have a cup of tea Facts: A, a doctor, conducted a together on a way side restaurant. hysterectomy sincerely on B and left a Sometime later, 'S' got a message from small cotton swab inside the abdomen. As his office to report back as it was long a consequence of which B developed time since he left the office. 'S' rushed some medical problems and had to back on his Motor Cycle. On his way back undergo another surgery. Is A liable? a Truck which was coming from a side A. Liability for negligence does not arise road hit 'S'. He was admitted in a nearby here as A performed the operation hospital with multiple injuries. He claims sincerely compensation under the Employees B. A is not liable as he did not foresee any Compensation Act from his employer. consequences at the time of surgery. A. The Employer is liable to pay C. As only a small swab was left in the compensation as the accident took place abdomen, there was no negligence.

14 www.gradeup.co

arising out of and in the course of Rahul, along with the Company's employment. products, used to carry Water Purifiers B. The Employer is not liable as the truck manufactured by his Cousin in a local driver was negligent. Industrial Estate. He used to sell the local C. The Employer is liable as S had to rush product at a lower rate giving the back to the office, because of the impression to the buyers that he is message from the office. offering a discount on the Company's D. The Employer is not liable as he was product. The Company Management admitted in a private hospital and not a detected the fraudulent activity of Rahul Government Hospital. and dismissed him from service. Rahul still continued to carry on with his activity 112.Principle: According to Sec. 2 of the of selling the local product pretending Industrial Disputes Act, 1947, 'Industrial that he was still a salesman of the dispute means any dispute or difference Company. Several customers got cheated between employers and employers or in this process. The fraud was noticed by between employers and workmen or the Company when the customers began between workmen and workmen, which is to complain about the product. The connected with the employment or non- customers demanded the Company to employment or the terms of employment compensate their loss. or with the conditions of labour of any A. The Company is not liable as Rahul was person'. dismissed by the Company. Facts: The employees of DK Enterprises B. The liability rests with the local met the management and requested half manufacturer as it was a defective a day leave to allow them to celebrate a product. lunar eclipse, which was going to happen C. The Company is liable to the customers two days later. The management refused who purchased the local product from the request. Does this situation amount Rahul only till he remained as a salesman to an 'industrial dispute'? of the Company. A. Yes, because there is some difference D. The Company is liable to compensate of opinion it would be an industrial all the customers as it did not inform the dispute. public about Rahul's fraudulent conduct B. No as declaring holidays is a and the subsequent dismissal. prerogative of the employer. So no industrial dispute. 114.Principle: An offer made by one C. As the difference of opinion between party when accepted by another makes it the employees and employer is on a contract. declaration of holiday it amounts to an Transactions: issue connected with employment or with 1) P offered to sell his house for Rs. 20 the terms of employment and hence, an lakhs to R; R told P that he was interested industrial dispute. to buy a house for 15 lakhs D. No as Lunar eclipse is unconnected 2) C was looking for a house for not more with employment. than 25 lakhs; P informed C that his house was available for 20 lakhs. 113.Principle: A master shall be liable 3) K wanted to buy some old furniture; L for the fraudulent acts of his servants told K that he would sell his furniture for committed in the course of employment. Rs. 10, 000. However, the master and third parties 4) R advertised to sell his old car for a must exercise reasonable care in this price of Rs. Three lakhs; S found the regard. advertisement and offered to buy it for Facts: Rahul was a door to door Rs. 2 lakhs 50 thousand; R agrees to sell salesman with United Manufacturing it to S. Company (the Company). The Company Which among the above is actually a was manufacturing Water Purifiers. contract?

15 www.gradeup.co

A. Situations 1 and 2 are contracts A. The Bank cannot initiate any action B. Situation 3 only is a contract against Soloman as he is the Secretary of C. Situation 4 only is a contract a Registered Trade Union. D. Situations 2 and 4 are contracts B. The Bank can recover the loan amount from the Trade Union as Soloman is the 115.Principle: Every agreement, by Secretary of the Union. which any party is restricted absolutely C. As Soloman did not use the loan from enforcing his right in respect of any amount for his use and hence, no action contract, by the usual legal proceedings can be initiated against him. in the ordinary Tribunals, is void to that D. The Bank can file a suit for recovery of extent. The law also provides that nobody the loan amount against Soloman as he can confer jurisdiction to a civil court by took the loan for a personal purpose and an agreement between parties. in such case no immunity will work. Facts: A and B entered into a valid contract for rendering certain service. A 117.Principle: Section 34 of Indian Penal clause in the contract was that in case of Code provides that 'When a criminal act any dispute arose out of the contract; it is done by several persons in furtherance shall be referred to for Arbitration only. Is of the common intention of all, each of the contract valid? such persons is liable for that act in the A. The contract is valid but the clause same manner as if it were done by him regarding Arbitration is void. alone B. Arbitration is also a valid dispute Facts: Three vagabonds, Sanju, Dilbag settlement machinery recognized by law and Sushil decided to commit burglary. In and hence the entire contract is valid. the night, Sushil opened the lock and they C. The parties were trying to confer broke into a rich man's house when the jurisdiction to some authority to decide a entire family was on a pilgrimage. Sanju dispute and hence the clause would be had gone to that house earlier in invalid. connection with some cleaning job. There D. Arbitrator cannot be termed as an was only a servant lady in the house. ordinary Tribunal. Hence, the agreement Hearing some sounds from the master is void and would be unenforceable. bed room, the servant switched on the lights and went up to the room from 116.Principle: According to the law of where she heard the sound. Noticing that trade unions in India, no suit or other the servant was going to cry for help, legal proceeding shall be maintainable in Sanju grabbed her and covered her any civil court against any registered mouth with his hands and dragged her trade union or any officer or member into the nearby room. The other two were thereof in respect of any act done in collecting whatever they could from the contemplation or in furtherance of a trade room. When they were ready to go out of dispute. the house, they looked for Sanju and Facts: Soloman, the Secretary of a found him committing rape on the registered Trade Union took a loan from servant. They all left the house and the a Bank for the higher education of his servant reported the matter to the police daughter. Soon after completing the and identified Sanju. Subsequently, all course she was married to an NRI three were arrested in connection with Engineer. Solomon did not repay the the offences of house breaking, burglary loan. The Bank demanded the payments and rape. Identify the legal liability of the from Soloman and warned him that the three. Bank will take suitable legal action A. All three are liable for all the offences against him. Identify the legal position in as there was common intention to commit this regard. the crimes. B. Only Dilbag and Sushil are liable for burglary in looting the house, and all

16 www.gradeup.co

three will be liable for housebreaking and 119.Principle: The Constitution of India rape as they did not stop Sanju from guarantees certain fundamental rights to committing the offence and hence were its citizens. The Constitution also provides accomplice to the offence. that these rights cannot be taken away by C. Only Sanju will be liable for rape as he state even by a law. For violation of this, was the one who actually committed the the person adversely affected by the law offence. may approach the High Court or the D. Sanju will be liable only for Supreme Court for the issuance of an housebreaking and rape as he did not appropriate writ. One of these rights participate in the burglary. includes the freedom to form association that implies the right to join an 118.Principle: In criminal law, association or not to join such an misappropriation is the intentional, illegal association. use of the property or funds of another Facts: Owing to some industrial person for one's own use or other disturbances created by XATU, one of the unauthorised purpose, particularly by a several trade unions in AB Chemicals public official, a trustee of a trust, an (Pvt) Ltd., the Company issued a circular executor or administrator of a dead to all its employees that as far as possible person's estate or by any person with a the employees may disassociate with responsibility to care for and protect XATU. Navin is an employee of AB another's assets. Embezzlement is Chemicals and the current General misappropriation when the funds Secretary of XATU. Aggrieved by this involved have been lawfully entrusted to circular, which affected the fundamental the embezzler. On the contrary, theft is rights of his and other members of the the illegal taking of another person's Union, approaches the High Court of the property or services without that person's state for a relief. Identify the most permission or consent with the intent to reasonable legal proposition. deprive the rightful owner of it. A. The circular interferes with the Facts: A went for swimming at the freedom guaranteed by the Constitution Municipal Swimming Pool. A handed over and hence the High Court can issue an all his valuables, including some cash to appropriate writ. X, the guard on duty for safe custody, as B. The Company's circular is illegal and notified by the Municipality. After has to be quashed by the Court. swimming for an hour, A came out and C. The prohibition against any imposition searched for X. He found another guard of restriction against a fundamental right on duty and that guard informed A that X is not applicable to anybody other than had gone home after completing his shift the state and hence Navin will not get any and did not hand over anything to be relief from the High Court. given to A. A registered a complaint with D. Circular issued by a Company amounts the police. X was traced but he told the to law in the constitutional sense and police that he sold all the valuables and hence the High Court can issue a writ as the entire cash was used for drinking pleaded for by Navin. liquor. What offence, if any, was /were committed by X? 120.Principle: There are legal provisions A. X is liable for theft as he took A's to give authority to a person to use property without X's permission. necessary force against an assailant or B. X is liable for criminal misappropriation wrong-doer for the purpose of protecting and embezzlement. one's own body and property as also C. X is not guilty of criminal another's body and property when misappropriation as he did not make any immediate aid from the state machinery personal gain out of those items with him. is not readily available; and in so doing D. If at all X is liable, it is for criminal he is not answerable in law for his deeds. misappropriation only.

17 www.gradeup.co

Facts: X, a rich man was taking his is not competent to contract, nothing in morning walk. Due to the threat of the Contract Act prevents him from robbers in the locality, he was carrying making the other party bound to the his pistol also. From the opposite minor. direction, another person was coming Facts: Lai executed a promissory note in with a ferocious looking dog. All of a favour of Gurudutt, aged 16 years stating sudden, the dog which was on a chain that he would pay Gurudutt a sum of Rs. held by the owner, started barking at X. 2 Lakhs when he attains the age of The owner of the dog called the dog to be majority. On attaining the age of 18, calm. They crossed each other without Gurudutt demanded the amount from Lai, any problem. But suddenly, the dog who refused to pay. Gurudutt wants to started barking again from a distance. X take legal action against Lai. Identify the immediately took out his pistol. By seeing most appropriate legal position from the the pistol the dog stopped barking and following: started walking with the owner. However, A. Lai argues that as per the Guardians X shot at the dog which died instantly. and Wards Act, 1890, Gurudutt can claim The owner of the dog files a complaint the money only after he attains the age against X, which in due course reached of 21. the Magistrate Court. X pleads the right B. Gurudutt should not have entered into of private defence. Decide. a contract with Lai when he was a minor. A. Shooting a fierce dog is not to be C. Lai was not aware of the fact that brought under the criminal law. So the Gurudutt was a minor. case should be dismissed. D. A promissory note duly executed in B. There was no imminent danger to X as favour of a minor is not void and can be the dog stopped barking and was walking sued upon by him, because he though with the owner. Hence, shooting it incompetent to contract, may yet accept amounted to excessive use of the right of a benefit. private defence and hence liable for killing the dog. 122.Principle: According to law, a C. As there was no guarantee that the person who find goods belonging to dog would not bark again, shooting it was another and takes them into his custody, a precautionary measure and hence is subject to the same responsibility as a within the right available to X under law. bailee. Bailee is a person or party to D. The right of private defence is available whom goods are delivered for a purpose, to persons against assailants or wrong- such as custody or repair, without doers only and a dog does not fall in this transfer of ownership. The finder of the category. goods legally can sell the goods found by him under certain circumstances 121.Principle: According to law, a including the situation that the owner person is deemed to have attained the refuses to pay the lawful charges of the age of majority when he completes the finder. age of 18 years, except in the case of a Facts: P, a college student, while coming person where a guardian of a minor's out of a Cricket stadium found a necklace, person or property has been appointed studded with apparently precious under the Guardians and Wards Act, 1890 diamonds. P kept it for two days thinking or where the superintendence of a that the owner would notify it in a local minor's property is assumed by a Court of newspaper. Since he did not notice any Wards. Indian law expressly forbids a such notification, P published a small minor from entering into a contract. classified advertisement in a local Hence, any contract entered into by a newspaper. In two days' time, P was minor is void-ab-initio regardless of contacted by a film actor claiming that it whether the other party was aware of his was her Necklace and requested P to minority or not. Further, though a minor return it to her. P told her that she should

18 www.gradeup.co

compensate him for the advertisement Coaching Centre closer to ABCC which charges then only he would return it resulted in a substantial drop in its otherwise he will sell it and make good his students and huge financial loss. The expenses. The film star told P that she owner of ABCC wants to file a case had advertised in a national newspaper against T for the loss sustained by ABCC. about her lost Necklace which was lost What do you think is the right legal somewhere in the Cricket Stadium. The position? advertisement was published for three A. T will be liable to compensate the loss consecutive days incurring a large to ABCC. expenditure for her. Mentioning all this B. T has not violated any of ABCCs legal she refuses to pay P and claims the right though they sustained some Necklace back. Which among the financial loss, and not legally bound to following is the most appropriate answer compensate ABCC. to this? C. T should have consulted ABCC before A. As the film star had notified in the starting his coaching centre. newspaper, P ought to have read it and D. T started the new coaching centre near contacted her instead of publishing ABCC intentionally, and shall be liable to another notification. So he cannot claim compensate the loss of ABCC. any compensation. B. P was requesting the film star for the 124.Principle: When a person falsifies actual expenditure incurred by him before something with the intent to deceive returning the Necklace. This request is another person or entity is forgery and is legally sustainable. a criminal act. Changing or adding the C. As it was wrong on the part of P to signature on a document, deleting it, bargain over a property belonging to a using or possessing the false writing is celebrity and he should have accepted also considered forgery. In the case of some gift which might have been given writing/painting to fall under the by the film star and returned the Necklace definition, the material included must instead of threatening her that he would have been fabricated or altered sell it. significantly in order to represent D. The film star was right in refusing P, as something it is actually not. she did not offer any reward for anyone Facts: David made a living traveling from who would return the Necklace. city to city, selling paintings that he claimed were done by great artists. Since 123.Principle: A violation of a legal right the artists' signatures were in place, of someone, whether results in a legal many people fell for them and purchased injury or not, gives rise to an action in tort the paintings. One of these artists saw for compensation. At the same time, an three of his alleged paintings in a City action by someone, which results in some gallery containing his name. He knew loss or damage to somebody else is not these were not his works and he actionable, if there is no violation of a complained to the police. Police traced right of that somebody. David and initiated legal proceedings. Is Facts: AB Coaching Centre was a popular David guilty of any offence? CLAT coaching academy with several A. David is guilty of forgery as the good trainers. A lot of aspirants used to addition of the signature was with an attend its coaching classes from all over intention to make people believe that and was making good profit. This was those were the paintings of the great going on for the past several years. artists. During a session, T, one of the very good B. David is not guilty of any offence as he and popular trainers of ABCC, had some was selling the art pieces for his living. difference of opinion with the owner of C. Those who buy the art pieces from ABCC and left the coaching centre. In David ought to have been careful in August 2016, T started another Entrance

19 www.gradeup.co

checking it and ensuring that they were Direction: Legal phrases are followed by originals before purchasing it. four meanings. Choose the most D. There is no point in taking legal action appropriate option against David as the signature has not 126. done any alteration to the art work. Bona vacantia A. Vacant land 125.Principle: When a person falsifies B. Goods that have no owner something with the intent to deceive C. Vacant building another person or entity is forgery and is D. Order of the court for eviction a criminal act. Changing or adding the signature on a document, deleting it, Direction: Legal phrases are followed by using or possessing the false writing is four meanings. Choose the most also considered forgery. In the case of appropriate option writing to fall under the definition, the 127. Caveat venditor material included must have been A. Buyer beware fabricated or altered significantly in order B. Manufacturer beware to represent something it is actually not. C. Seller beware Facts: John was a publisher of ancient D. Transporter beware books and papers. In one of his books on the World Wars, he gave photograph of Direction: Legal phrases are followed by some letters written by famous historic four meanings. Choose the most personalities. A researcher in history appropriate option noted that in the pictures of some of the 128. Faux pas letters printed in the book, John had A. Tactless mistake added some words or sentences in his B. Cheating own handwriting to give completeness to C. Pausing for a while the sentences, so that the readers will get D. Passage of time a clear picture of the writer's intention. The researcher challenges the originality Direction: Legal phrases are followed by of those pictures and claims that the book four meanings. Choose the most containing the forged letters should be appropriate option banned. Examine the validity of the 129. In pari delicto researcher's demand. A. Where the lawyer is at fault A. As forgery amounts to adding or B. Where the judge is at fault deleting anything from an original C. Where the petitioner is at fault document, the demand of the researcher D. Where both parties to a dispute are is valid. equally at fault B. Allowing forged publications to be circulated among the public is as good as Direction: Legal phrases are followed by committing fraud on the public, so the four meanings. Choose the most publication should be banned. appropriate option C. The additions in the letters were made 130. Lex loci by the publisher in his own handwriting A. Law of a place would have made material alteration to B. Latin regualtions the original meaning and hence C. Italian laws amounted to forgery. D. Domestic laws D. The additions were made to give clarity to the original document and did not in Direction: Legal phrases are followed by any sense change the contents of the four meanings. Choose the most documents and hence there is no forgery appropriate option as alleged by the researcher. 131. Malus animus A. Animal farm B. Bad intention

20 www.gradeup.co

C. Good intention Direction: Legal phrases are followed by D. Physical force four meanings. Choose the most appropriate option Direction: Legal phrases are followed by 137. 'Punctum Temporis' means: four meanings. Choose the most A. Temporary position appropriate option B. Point of time 132. Per incuriam C. Functional authority A. Supremacy of law D. Timely assistance B. Supremacy of the Constitution C. Mistaken decision Direction: Legal phrases are followed by D. Mistaken identity four meanings. Choose the most appropriate option Direction: Legal phrases are followed by 138. Turpis arbiter' means: four meanings. Choose the most A. Corrupt prosecutor appropriate option B. Inefficient judge 133. Pari passu C. Corrupt judge A. On an unequal status D. Inefficient lawyer B. Supremacy of law C. Diverse nature Direction: Legal phrases are followed by D. On equal footing four meanings. Choose the most appropriate option Direction: Legal phrases are followed by 139. 'Jus Gentium' means: four meanings. Choose the most A. Global administrative law appropriate option B. Law of Societies 134. Autrefois convict C. Law among Nations A. Formerly convicted D. Global justice B. To be convicted C. Failed prosecution Direction: Legal phrases are followed by D. Doubtful conviction four meanings. Choose the most appropriate option Direction: Legal phrases are followed by 140. ‘Animus posssidendi' means: four meanings. Choose the most A. Intention to return appropriate option B. Intention to harm 135. Lis pendens C. Intent to contract A. No legal issues involved D. Intention to possess B. Facts of case proved C. Pending suit Direction: Read the following D. Decided case information carefully and choose the appropriate option in the questions given Direction: Legal phrases are followed by below. four meanings. Choose the most i. There is a group of five persons - A, B, appropriate option C, D and E 136. 'Sine die' means: ii. One of them is a Singer, one is a A. Adjourned for the day and scheduled Dancer, one is a Painter, one is a Teacher to meet next day again. and one is a Doctor. B. Adjourned for the day and meet after iii. Three of them - A, C and Doctor prefer one week. rice to chapatti and two of them - B and C. Adjourned without fixing any date for the Painter prefer chapatti to rice. the next meeting. iv. The Teacher, D and A are friends to D. Adjourned for the day and meet after one another but two of these prefer one month. chapatti to rice. v. The Singer is C's brother.

21 www.gradeup.co

141. Directions (147 – 148) : Which Who is a Singer? alternative applies to the following A. A Statement or Assumptions? Choose the B. B most appropriate option. C. C 147. D. D 'There is no man that is not naturally good' is equivalent to the proposition: 142.Who is a Teacher? A. Some men are naturally good. A. E B. Some men are not naturally good. B. C C. No men are good. C. B D. All men are naturally good D. D 148.'Only ignorant people believe in 143.Who is a Dancer? witchcraft' is equivalent to: A. A A. All persons who believe in witchcraft B. C are ignorant. C. D B. No ignorant persons are those who do D. E not believe in witchcraft. C. Some ignorant persons are not those Direction (144 – 146) : Read the who believe in witchcraft. following information carefully and D. There is no link between ignorance and choose the most appropriate option in the witchcraft. questions given below: i. Six flats on a floor in two rows facing 149. North and South are allotted to P, Q, R, Find the odd one out from the following: S, T and U. A. Exact estimate ii. Q gets a North-facing flat and is not B. Only choice next to S. S and U get diagonally opposite C. Clearly visible flats. D. Open secret iv. R, next to U gets a South-facing flat and T gets a North-facing flat. 150.Find the odd one out from the following: 144. A. Expedition Which of the following combination gets B. Cruise South-facing flats? C. Crusade A. Q, T, S D. Campaign B. U, P J C. U, R, P Directions (151 – 152) : Choose the D. data inadequate most appropriate option for each of the following questions. 145.Whose flat is between Q and S? 151. A. T Pointing to a photograph, Prakash said, B. U 'She is the daughter of my grandfather's C. R only son. How is Prakash related to the D. P girl in the photograph? A. Father 146.If the flats of T and P is interchanged, B. Brother who's flat will be next to that of U? C. Uncle A. P D. Cousin B. Q C. T 152.When Ravi saw Ramesh, he recalled, D. R 'He is the son of the father of my daughter'. Who is Ramesh?

22 www.gradeup.co

A. Brother-in-law C. Some parrots are not crows. B. Brother D. No conclusion can be drawn. C. Cousin D. Son 158.Pointing to a girl in the photograph, Ram said, 'Her mother's brother is the 153.Sunil's school bus is facing North only son of my mother's father'. How is when it reaches his school. After starting the girl' s mother related to Ram? from Sunil's house, it turned right twice A. Mother and then left before reaching the school. B. Sister What direction the bus was facing when it C. Aunt left the bus stop in front of Sunil's house? D. Cannot be determined A. East B. North 159.A girl introduced a boy as the son of C. South the daughter of the father of her uncle. D. West The boy is girl's: A. Son-in-law 154.John wants to go the university. He B. Son starts from his house which is in the East C. Uncle and comes to a crossing. The road to his D. Brother left ends in a theatre, straight ahead is the hospital. In which direction is the 160.If South-East becomes North, North- University? East becomes west and so on, what will A. East West become? B. North A. South-East C. South B. East D. Cannot be determined C. North D. North-West 155.Two ladies and two men are playing bridge and seated at North, East, South 161.A man walks 1 km. towards East and and West of a table. No lady is facing then he turns to South and walks 5 kms. East. Persons sitting opposite to each Again he turns to East and walks 2 kms. other are not of the same sex. One man After this he turns to North and walks 9 is facing South. Which direction are the kms. Now, how far is he from his starting ladies facing to? point? A. North and West A. 10 kms. B. East and West B. 9 kms. C. South and East C. 4 kms. D. None of these D. 5 kms.

156.'Some of the valuable books are 162.Vaishnavi prefers Economics to seldom read', means: Math’s, English to Social science, and A. All the valuable books are not read. Political Science to History. If she prefers B. Some of the valuable books are read. Math’s to History, and Social science to C. Some of the valuable books are not Math’s, which is Vaishnavi's least read. preferred D. All the valuable books are read. A. History B. Math’s 157.No parrots are black. C. Social science All crows are black. D. Economics From the above premises which one of the following conclusions is true? 163.Wave: crest as ____: peak. A. Some crows are not parrots. A. Mountain B. No crows are parrots. B. Water

23 www.gradeup.co

C. River photograph, Ravi said: 'The boy sitting at D. Land the left is the son of the wife of the only son of the grand-mother of my younger 164.Crumb: Bread is as brother'. A. Inch: Unit What is the relation between the boy in B. Water: Vessel the photograph and Ravi? C. Splinter: Wood A. First Cousins D. Powder: Face B. Brothers C. Ravi's brother-in-law 165.A person who renounces religious or D. Nephew and uncle political belief or principle is called: A. Apostle 171.In a military secret service map, B. Antiquarian South-East is shown as North, North-East C. Ascetic as West and so on. What will West D. Apostate become? A. South-East 166.______is a hater of knowledge and B. North-East learning. C. South-West A. Misogynist D. North-West B. Mystique C. Misologist 172.The birthday of Ms. Y was celebrated D. Moroccan six days before Ms. X, who was born on 4th October 1999. The Independence Day 167.What is meant by 'Alliteration'? of that year fell on Sunday. On which day A. Words which sound alike but have did Ms. Y celebrate her birthday, if it was different meanings. not a leap year? B. The occurrence of the same letter or A. Tuesday sound at the beginning of adjacent or B. Monday closely connected words. C. Sunday C. Acts of an environmentally conscious D. Wednesday person. D. Act of literary modification. 173.In the series of alphabets given below, which is the missing alphabet 168.If in a code language, 'ABANDON' is series? written as 'aramoim'; 'BORE' is written as AX, DU, GR, ………. ML 'rits' and 'BASIL' is written as 'rabut', then A. JN what is the original word for the code: B. HO 'bituo'? C. IK A. SOFIA D. JO B. SOLID C. NASIA 174.Mare is to Horse as - D. SOMAD A. Deer is to Buck B. Sow is to Boar 169.How many times from 4 pm to 10 C. Pony is to Donkey pm, the hands of a clock are at right D. Geese is to Duck angles? A. 6 175.Coding and decoding 9: 72: : 8 : ? B. 11 A. 34 C. 10 B. 64 D. 9 C. 18 D. 43 170.Ravi was showing a photograph to his friend, Gopi. Pointing at a boy in the

24 www.gradeup.co

176.If in a certain code, the word In an office, 1/3 of the workers are MILITARY is written as 12324567, then in women, 1/2 of the women are married the same code, the word TAIL will be and 1/3 of the married women have written as: children. If 3/4 of the men are married A. 2345 and 2/3 of the married men have B. 4523 children, then the part of workers without C. 5432 children are: D. 3254 A. 4/9 B. 5/18 177.There were twelve dozens of C. 17/36 chocolates with a shopkeeper. Ten D. 11/18 chocolates were distributed by the shopkeeper to the children of his colony. 182.The average weight of three men 'X', The shopkeeper then added two more 'Y' and 'Z is 75 kgs. Another man 'A' joins dozens of chocolates in his stock. If the the group and the average weight now shopkeeper divided the total chocolates becomes 80 kgs. If another person 'B' equally in two different packets, then how whose weight is 5 kgs more than 'A' many chocolates were there in each replaces 'X', then the average weight of packet? 'Y, 'Z', 'A' and 'B' will be 85 kgs. What is A. 79 the weight of 'X'? B. 89 A. 78 kgs. C. 158 B. 80 kgs. D. 152 C. 82 kgs. D. 84 kgs. 178.In a company, 60% workers are males. If the number of female workers 183.'A’ and 'B' complete a work in 12 in the company is 800, what is the days, 'B' and 'C’ in 8 days and 'C' and 'A’ number of male workers in the company? in 16 days. 'A' left after working for 3 A. 1900 days. In how many days more will 'B' and B. 1200 'C’ finish the remaining work? C. 1400 A. 3¾ D. 1600 B. 4¾

179.If 27th March, 2011 was Sunday, C. 6 what was the day on 27th June, 2011? D. 7¾ A. Sunday B. Monday 184.A train X leaves station 'A' at 3.00 C. Saturday p.m and reaches station 'B' at 4.30 p.m., D. Tuesday while another train T leaves station 'B' at 3.00 p.m and reaches station 'A' at 4.00 180.Identify the statement which cannot p.m. These two trains cross each other be accepted at: A. Odyssey is an ancient epic A. 3.20 p.m. B. Human race will become extinct sooner B. 3.30 p.m. or later C. 3.36 p.m. C. Almost one third of the human body is D. 3.40 p.m. made up of water D. The earth revolves around the sun in 185.A vessel contains a mixture of milk 366 days and water in the ratio of 5 : 3 respectively. How much of the mixture Direction (181 – 200) : Choose the must be siphoned off and replaced with most appropriate option. water, so that the mixture may be half 181. milk and half water?

25 www.gradeup.co

A. 1/3 C. 12% B. 1/4 D. 10% C. 1/5 D. 1/7 191.Praveen has Rs. 4,662 in the form of 2, 5 and 10 rupee notes. If these notes 186.Taps 'A' and 'B' can fill a tank in 37 are in the ratio of 3 : 5 : 8, then the number of five rupees notes with him is: minutes and 45 minutes respectively. A. 84 Both taps are opened and after some time B. 210 tap 'B' is turned off. The tank is filled C. 250 completely in exactly 30 minutes, if tap D. 336 'B' is turned off after: A. 15 minutes 192.A clock was set correct at 12 O' clock. B. 12 minutes It loses 10 minutes per hour. What will be C. 10 minutes the angle between the hour and minute D. 9 minutes hands of the clock after one hour? A. 105° 187.A man rows to a place 45 k.ms B. 90° distant and back in 12 hours. He realizes C. 85° that he can row 5 k.ms downstream in the D. 75° same time as 3 k.ms against the stream. The velocity of the stream is: 193.The difference between simple A. 2 k.ms/hr interest and compound interest at the B. 1.5 k.ms/hr same rate for rupees 5,000 for two years C. 1 k.m/hr is rupees 98. The rate of interest is: D. 4 k.ms/hr A. 14% B. 12% 188.A trader sells rice at a profit of 20% C. 10 ½ % and uses weights which are 10% less D. 10% than the correct weight. The total gain earned by him is: 194.Gold and copper are as heavy as A. 22 2/9% water by 19 and 9 times respectively. The B. 30% ratio in which these two metals be mixed C. 33 1/3% so that the mixture is 17 times as heavy D. 35% as water is: A. 3:4 189.A piece of cloth costs rupees 75. If B. 4:1 the piece is four meters longer and each C. 2:3 meter costs rupees 5 less, the cost D. 3:2 remains unchanged. What is the length of the piece? 195.A can do a piece of work in 8 days A. 12 meters and B alone can do the same work in 10 B. 10 meters days. A and B agreed to do the work C. 8 meters together for Rs. 720. With the help of C, D. 6 meters they finished the work in 4 days. How much C is to be paid? 190.The Banker's discount on a sum of A. Rs. 70 money for 18 months is Rs. 600 and the B. Rs. 82 true discount on the same sum for 3 C. Rs. 80 years is Rs. 750/-. The rate percentage D. Rs. 72 is: A. 20% 196.Two men and seven boys can do a B. 15% work in 14 days. Three men and eight

26 www.gradeup.co

boys can do the same work in 11 days. D. 1/2 Further eight men and six boys can do three times the amount of this work in: 199.Age of father 10 years ago was three A. 18 days times the age of his son. After 10 years, B. 21 days father's age is twice that of his son. The C. 24 days ratio of their present ages is: D. 30 days A. 11 : 7 B. 9 : 5 197.A boat travels upstream from A to B C. 7 : 4 and back from B to A in 5 hours. The D. 7 : 3 speed of the boat in still water is 8 km/hour and the speed of the current is 200.Keerthi's father gave him some 4 km/hour. Then, the distance from A to money to buy books. He spent half of the B is: money equally to buy books and A. 9 kms. entertaining his friends. Whatever B. 10 kms. amount left with him, he deposited half in C. 12 kms. his savings account and gave Rs. 5 to a D. 15 kms poor person as charity. Finally, Keerthi was left with Rs. 20 which he returned to 198.There are two urns. One contains two his father. What amount did his father white balls and four red balls, the other give him initially? contains three white and nine red balls. A. RS. 200 All balls are of the same shape and size. B. RS. 160 From each urn, one ball is drawn. What is C. Rs. 100 the probability of getting both the balls of D. RS. 120 the same colour? A. 7/12 B. 1/12 C. 1/24

27 www.gradeup.co

###ANSWERS### 1. Ans. B. The adverb ‘but’ means ‘no other than’. The sentence implies that to succeed, we Therefore, ‘but’ is the correct fit for the need to be diligent and hard-working. blank. Also, the sentence needs a conjunction Rather is used to indicate one's which means ‘except if’. ‘Unless’ means preference in a particular matter. ‘except if (used to introduce the case in which a statement being made is not true 8. Ans. C. or valid)’. The sentence implies that the passengers Therefore, unless is the correct fit for the were very happy with respect to the blank. friendly and warm treatment. The adverb 2. Ans. A. ‘about’ meaning ‘on the subject of; The context of the sentence is that ‘She’ concerning’ is the correct fit for the blank. was standing in front of Amit. The correct 9. Ans. C. preposition to imply this is ‘before’ which The sentence is an example of first means in front of someone/something. conditional sentences. The type 1 Therefore, ‘before’ is the correct fit for conditional is used to refer to the present the blank. or future where the situation is real. The 3. Ans. B. type 1 conditional refers to a possible The verb ‘endowed’ is always followed by condition and its probable result. We use the preposition ‘with’. Without a the simple present tense in the if-clause preposition, ‘endow’ means ‘to provide and simple future tense (will + infinitive) with something freely or naturally’. in the main clause—that is, the clause 4. Ans. B. that expresses the likely outcome. The sentence implies that the minister Therefore, will is the correct fit for the flew above the flooded areas in a blank. helicopter. The preposition ‘over’ means 10. Ans. C. ‘at a higher level or layer than’. The phrase ‘set-aside’ means ‘to save Therefore, over is the correct fit for the for a particular purpose’. The sentence blank. implies that Sunita decided to save some Fly about means ‘circulate (rumours, etc)’ time every day for prayers. Therefore, 5. Ans. A. aside is the correct fit for the blank. The sentence implies that the person has 11. Ans. A. played a great role in the subject’s life Refer to the following sentence from the because in his absence, the subject would passage: ‘Finally, and perhaps most have landed himself into a problem. important, many countries at the ‘Without’ means ‘in the absence of’. Uruguay Round came to put a higher Therefore, option a is the correct priority on the export gains than on the answer. import losses that the negotiation would 6. Ans. C. produce, and they came to associate the The phrase ‘go through’ means ‘search WTO and a rule-based system with through or examine methodically’. The those gains.’ Here, “it” refers to ‘the sentence aptly provides a meaning when export gains many countries came to the blank is filled with ‘through’, hence, associate with a rule-based system’. making option c the correct answer. Therefore, option a is the correct The phrase ‘go under’ means ‘(of a answer. business) become bankrupt’. 12. Ans. B. 7. Ans. A. According to the passage, ‘For the United The sentence implies that apart from States, which did not want a new mother, nobody would have risked her organization, the disputed settlement life for the baby. ...’. This provides the reason why WTO was not formed in 1970s. So, since the important players did not find it in their

28 www.gradeup.co

best interest to do so, WTO was not option b should replace the underlined formed in 1970s. Therefore, option b is part. the correct answer. 17. Ans. A. 13. Ans. A. The underlined fragment should be Refer to the following sentence from the replaced with option a to maintain passage: ‘One means the Court used to parallelism. Parallelism is the use of expand integration was the 'teleological components in a sentence that are method of interpretation', whereby the grammatically the same; or similar in actions of member states were evaluated their construction, sound, meaning, or against 'the accomplishment of the most meter. "As well as" sets apart an item as elementary goals set forth in the different from / unequal to the others, Preamble to the (Rome) treaty’’. It thus its usage is incorrect here. Hence, suggests that actions against member option a should replace the underlined states needed to be evaluated against the part. said community goals according to the 18. Ans. B. method of interpretation of the European ‘Though’ meaning ‘despite the fact that; Court of Justice. although’ can be used in the beginning of Therefore, option a is the correct the sentence to provide the same answer. meaning as that of the underlined 14. Ans. A. fragment. Out of option b and d, option According to the passage, ‘As it played b is the correct one because we need a out in the WTO, consistency meant subject which should be modified by the integrating under one roof the whole adjective ‘plausible’. lot of separate agreements signed 19. Ans. B. under GATT auspices…’. This suggests Option a is incorrect because of the usage that WTO promoted the technical legal of ‘which’ is the sentence. Options c and values partly through integrating under d are not grammatically correct with one roof the agreements signed under regard to sentence structure. Option b GATT. aptly rephrases the underlined sentence. Therefore, option a is the correct 20. Ans. D. answer. The underlined part must be replaced 15. Ans. D. with ‘For not everyone attending knew’ to According to the passage, ‘…the WTO render the correct meaning. We need a package was acceptable as a means to main verb after the auxiliary one, ‘was’, discipline the resort to unilateral for which the appropriate form is ‘knew’. measures by the United States. Countries 21. Ans. B. like Canada and other middle and smaller The correct spelling is accommodation. trading partners were attracted by the It means ‘a room, group of rooms, or expansion of…’. This suggests that the building in which someone may live or need for a rule-based environment to stay’. protect the benefits of increased trade 22. Ans. B. was the most likely reason for the The correct spelling is gallows. It means acceptance of the WTO package by ‘a structure, typically of two uprights and nations. a crosspiece, for the hanging of Hence, option d is the correct answer. criminals’. 16. Ans. B. 23. Ans. B. The subject in the sentence is ‘the other’ The correct spelling is remuneration. It which is singular, hence, the verb must means ‘money paid for work or a service’. also be singular. Therefore, the verbs 24. Ans. D. must be ‘occurs and contributes’ ruling The correct spelling is blasphemy. It out options a and c. Also, the correct means ‘the action or offence of speaking phrase is ‘one and the other’. Hence, sacrilegiously about God or sacred things; profane talk’.

29 www.gradeup.co

25. Ans. C. ‘At hand’ is erroneous. ‘At’ should be The correct spelling is hypothecation. replaced with ‘in’ as the phrase ‘in hand’ Hypothecation occurs when an asset is means ‘receiving or requiring immediate pledged as collateral to secure a loan, attention’. Therefore, option c is the without giving up title, possession or correct answer. ownership rights, such as income 35. Ans. C. generated by the asset. ‘Was purple’ is erroneous. It should be 26. Ans. A. ‘turned red’ to render the correct The correct spelling is gratuitous. It meaning. If you ‘go/turn red’, your face means ‘done without good reason; becomes red because you are uncalled for’. embarrassed. Therefore, option c is the 27. Ans. A. correct answer. The correct spelling is interrogation. It 36. Ans. B. means ‘the action of interrogating or the ‘Being finished’ has the error. process of being interrogated’. Prepositions must always be followed by 28. Ans. C. a noun or pronoun. So, we need to use The correct spelling is annulment. It ‘finishing’, the gerund form (acting as a means ‘the act of annulling something’. noun) after the preposition ‘after’ to make 29. Ans. C. the sentence correct. Therefore, option The correct spelling is abeyance. It b is the correct answer. means ‘a state of temporary disuse or 37. Ans. D. suspension’. ‘The scratch’ is erroneous. The phrase is 30. Ans. D. ‘start from scratch’. ‘To start from The correct spelling is moratorium. It scratch’ means to start from the means ‘a temporary prohibition of an beginning, to set out on some action or activity’. process without any prior preparation, 31. Ans. C. knowledge or advantage. Therefore, ‘Unique’ means one of a kind, one can't option d is the correct answer. really get degrees of uniqueness. Thus, 38. Ans. A. ‘the most’ is erroneous because ‘most’ ‘May visit’ is erroneous. It should be cannot be used with ‘unique’ as it makes replaced with ‘might visit’ since the main it superfluous use of the degree of clause is in Past Tense, so the adjectives. Therefore, option c is the subordinate clause should also be in past. correct answer. Contextually, ‘might’ suggests a lower 32. Ans. D. probability than ‘may’ does. Earlier, there ‘Lifting’ is erroneous because it violates was no chance in the absence of the the rule of parallelism. Parallelism is the ticket. Now, there is a remote one use of components in a sentence that are available. Therefore, option a is the grammatically the same; or similar in correct answer. their construction, sound, meaning, or 39. Ans. B. meter. ‘Are’ is erroneous’. When nouns Thus, bare infinitive form of the verb ‘lift’ expressing periods of time, amounts of i.e. ‘lift’ will be used in place of ‘lifting’ money or quantities are considered as a (gerund) to make the sentence correct. singular unit, singular verbs are used. Therefore, option d is the correct Therefore, ‘are’ must be replaced with ‘is’ answer. to make the sentence grammatically 33. Ans. A. correct. Part A has an error. ‘The hope’ must be Therefore, option b is the correct preceded by ‘with’ to render the correct answer. meaning. Therefore, option a is the 40. Ans. B. correct answer. ‘Be submitted’ has the error because 34. Ans. C. there must be a modal (must) before ‘be’ to make the sentence meaningful.

30 www.gradeup.co

Therefore, option b is the correct answer back to the prison campus after their 41. Ans. C. working hours. ● In February 2017 the Defence Research 43. Ans. C. and Development Organisation (DRDO) ● India’s first Commercial Court and successfully test fired a Prithvi Defence Commercial Disputes Resolution Centre Vehicle (PDV) interceptor missile was inaugurated at Raipur, capital of designed to intercept and destroy hostile Chhattisgarh. ballistic missiles in space even before ● It was inaugurated by State Chief they enter the Earth's atmosphere. Minister Dr. Raman Singh along with ● This further enhances India's capability Supreme Court judge Madan B. Lokur. of dealing with a nuclear attack threat. Note: ● Its interception window spans from 80- ● The court also consists of arbitration 120 km. centre and a mediation centre. ● The fully automated system consists of ● Some facilities available for litigants a network of sensors, computers and through the new system includes video- launchers, designed to intercept hostile conferencing, E-Court, E-Library, E-Filing ballistic missiles, possibly carrying and E-Summons. nuclear weapons and destroy them ● These ultra-modern facilities available before they can cause any damage. at the court will facilitate the investors in 42. Ans. A. their commercial dealings. ● Highest number of open prisons in India ● The Court will help in improving the as on 2015 are in Rajasthan. judicial system. ● Rajasthan which has as many as 29 44. Ans. A. open prisons, the highest in the country. ● The world’s first robot lawyer named ● Maharashtra (13) having the highest ‘ROSS’ has been employed by a US law number. firm BakerHostetler. ● As per the Rajasthan Prisoners open-air ● The first of its kind artificial intelligence camp rules, 1972 the Open Jails Air Camp (AI) lawyer will assist the law firm and its Rules, 1972 as follows: ‘prisons without various teams in legal research. walls, bars and locks.’ ● The robot was built by the company Note: ROSS Intelligence and its working is ● Open prisons refer to the prisons that based on cognitive computing. involve minimum security and is mainly Note: dependent on the self-discipline of the ● The ‘ROSS’ robot is built upon IBM’s inmates. cognitive computer Watson. ● The rules of these prisons are less ● It will function using Watson’s cognitive stringent as compared to the rules of the computing and natural language other prisons. processing capabilities. ● For this reason, they are also known as ● Lawyers can ask ROSS their research open air camps, prisons without bars or question and which it will answer by minimum security prison. reading through the law, gathered ● These promote one of the major evidence, drawn inferences. principles of punishment known as the ● ROSS will also monitor the law around ‘Reformative Theory’ where instead of the clock to notify users of new court severely punishing the convict, an decisions that can affect a case. opportunity is provided to reform oneself ● It is programmed to continually learn and get into the mainstream life. from the lawyers who will in turn use it to In open jails the prisoners are given bring back better results each time. the liberty: 45. Ans. B. ● To live with their families ● Surgeon Raghu Ram awarded 2016 B ● Allowed to find employment C Roy Award. ● Prisoners can move out of the prison for ● Dr P Raghu Ram, President of their work and are supposed to come Association of Breast Surgeons of India

31 www.gradeup.co

was awarded prestigious Dr B C Roy ● Its purpose is to take action to combat national award for outstanding service in corruption and prevent criminal activities the field of socio-medical relief for 2016. arising from corruption. ● He is claimed to be the youngest ● It publishes annually Corruption surgeon ever from the Telugu states Perceptions Index and Global Corruption (Andhra Pradesh and Telangana) to Barometer. receive this award. 47. Ans. C. About BC Roy Award ● Dalai Lama was called a 'deceptive ● The BC Roy Award is the highest actor' by China's foreign ministry during recognition for medical practitioners in March 2017. India. ● China’s Foreign Ministry called the Dalai ● It was instituted by the Medical Council Lama a “deceptive actor” after the exiled of India (MCI) in 1976 in memory of Tibetan spiritual leader said in an renowned physician and former West interview that Chinese hardliners have Bengal Chief Minister Dr Bidhan parts of their brains missing. Chandra Roy. ● The Dalai Lama, who fled into exile in ● The award is bestowed annually 6 India after a failed uprising against categories viz. statesmanship of the Chinese rule in 1959, denies espousing Highest Order in India, eminent medical violence and says he only wants genuine person, medical man-cum-statesman, autonomy for Tibet. eminent person in philosophy, eminent 48. Ans. C. person in arts and eminent person in ● Hungary's prime minister Viktor Orban Science. said that - Migration is the "Trojan 46. Ans. C. wooden horse" of terrorism and the ● According to recent survey released by current lull in the migrant flow is only Transparency International (TI), India temporary. had highest bribery rate among the 16 ● The people that come to us don't want Asia-Pacific countries surveyed between to live according to our culture and July 2015 and January 2017. customs but according to their own — at ● According to the survey, Police, a key European standards of living.'' law and order institution suffer most from ● Orban said the migration pressure on corruption. Nearly two in five people in Hungary's borders would continue as the region mentioned the police were millions of people were planning to come mostly or entirely corrupt (39%). to Europe in hope of better lives. Key Highlights of the Survey 49. Ans. C. ● Countries having highest incidence of ● India loses 11 billionaires after note bribery: 69% of the study group in India ban. said that they have paid a bribe, done a ● According to the Hurun Global Rich List favour or given a gift to receive services. India, the country has lost 11 billionaires ● follows this suit with 65%, since the note ban announced on Thailand 41% and Pakistan 40%. China November 8, 2016. reported a much lower 26%. ● However, the aggregate total wealth of ● Countries having lowest incidence of the ultra-rich people has increased by 16 bribery: Japan had the lowest incidence per cent over last year. of bribery at 0.2% followed by South 50. Ans. C. Korea and recorded 3% each, ● Donald Trump elected as the 45th 2% and Taiwan 6%. President of the United States. About Transparency International ● Donald John Trump is the 45th and (TI) current president of the United States. ● It is an international non-governmental ● Before entering politics, he was a organization based in Berlin, Germany. businessman and television personality. ● Trump was born and raised in the New York City borough of Queens, and

32 www.gradeup.co

received an economics degree from the ● In January 2017, Uttar Pradesh Wharton School. Governor Ram Naik disqualified BSP’s ● He co-authored several books, including Rasra MLA Uma Shankar Singh from The Art of the Deal. Assembly membership for violation of ● He owned the Miss Universe and Miss Representation of the People Act. USA beauty pageants from 1996 to 2015, ● Governor Ram Naik disqualified BSP and produced and hosted The Apprentice, MLA Uma Shankar Singh from his a reality television show, from 2003 to membership of the state Assembly for 2015. procuring government contracts in his 51. Ans. B. name. ● 'Scorpion kick' is a phrase used in ● The Governor has ordered the Kabaddi. disqualification of Uma Shankar Singh ● 'Scorpion kick' is a back kick. It is an from the membership under section 192 essential skill for a raider to kick (1) of the Constitution as per the opinion backwards. of the Election Commission (EC) received ● The raider bends down on one knee, on January 10. snaps the other leg back towards an Note: In the exam the first option is given opponent’s chest or face, and usually has wrong. The correct option should be Uma his hands on the mat ready to spring back Shankar Singh of Uttar Pradesh instead of to the mid-line. Bihar. ● It’s an important weapon in a raider’s 55. Ans. C. arsenal because it lets them go on a ● New Balance has won a record payout surprise attack while maintaining in a Chinese trademark case after three distance from the defenders. local shoemakers were found to have 52. Ans. C. infringed the brand's "N" logo. ● According to the Ministry of Commerce ● A Chinese court awarded the US and Industry, the fifteen largest trading sportswear firm more than 10 million partners of India represent 59.37% of yuan (£1.2m; $1.5m). total trade by India in the financial year ● Lawyers believe it to be the highest 2015-2016. award to a foreign company in a ● These figures include trade in goods and trademark dispute in China. commodities, but do not include services ● The country has been tightening its laws or foreign direct investment. to tackle the widespread problem of ● During 2016, India’s largest trade trademark abuse. partners China, US and UAE. 56. Ans. C. 53. Ans. A. ● World Bank cuts India’s GDP ● The Union Government has decided to growth forecast to 7% for FY17. set up a single permanent Tribunal to ● The World Bank has sharply cut its adjudicate all inter-State river water economic growth forecast for India to 7 disputes. per cent in 2016-17 after taking into ● This single body will subsume all account the impact of demonetisation and existing tribunals for resolving grievances the fall in private investments. of inter-State water disputes in a speedy 57. Ans. A. manner. It will be headed by a retired ● Mangaluru girl Srinidhi Shetty was Supreme Court judge. crowned at a ● The Tribunal will also have more teeth grand ceremony in Krynica Zdroj, . as whenever it gives order, the verdict ● She also won the title of Miss Asia- gets notified automatically. Until now, the Oceania. 's Valeria Vespoli was Union Government was required to notify placed second followed by 's the awards, causing delay in its Jaleesa Pigot. implementation. 58. Ans. B. 54. Ans. A. ● In 2014-15, India surpassed the US to become the third largest steel producer in

33 www.gradeup.co

the world. India continued with this arrested him from Balochistan on March position in 2015-16 as well. 3, 2016. ● The country produced 88.97 million Note: tonnes and 89.79 million tonnes of crude ● The headquarter of the International steel during 2014-15 and 2015-16, court of Justice (ICJ) is at the Peace respectively Palace in , . 59. Ans. C. ● The International Court of Justice is the ● In January 2017, Volvo has launched primary judicial branch of the United the world’s largest bus chassis at the Nations (UN). FetransRio exhibition in Rio de Janeiro, ● The court settles legal disputes . submitted to it by states and provides ● Volvo is a leading supplier of buses for advisory opinions on legal questions high passenger capacity transport submitted to it by duly authorized systems, Bus Rapid Transit (BRT). international branches, agencies, and the ● The new biarticulated chassis, Gran UN General Assembly. Artic 300, is 30 meter and can carry up ● ICJ was established in 1945. to 300 passengers. 62. Ans. B. 60. Ans. A. ● In December 2016, Japanese PM ● Japan is threatening to take India to the Shinzo Abe has visited the US naval WTO over restrictions that nearly halved base at Pearl Harbor, where he offered its steel exports to the South Asian nation "sincere and everlasting condolences" to over the past year, a step that could the victims of Japan's attack on the base trigger more trade spats as global 75 years ago. tensions over steel and other ● Mr Abe was accompanied by US commodities run high. President Barack Obama, making the visit ● Such action is rare for Japan. the first by the leaders of both countries. ● The world’s second-biggest steel ● Mr Abe is the first Japanese leader to producer typically tries to smooth visit the memorial on the site of the disputes quietly through bilateral talks, Arizona, although several of his but with global trade friction increasing, predecessors have been to Pearl Harbor Japan’s defence of an industry that sells in the past. nearly half of its products overseas is Note: getting more vigorous. ● Pearl Harbor is a U.S. naval base near 61. Ans. C. Honolulu, Hawaii, that was the scene of a ● In May 2017, the International Court devastating surprise attack by Japanese of Justice stayed the hanging of Indian forces on December 7, 1941. national Kulbhushan Jadhav, who was ● Just before 8 a.m. on that Sunday sentenced to death by a Pakistani military morning, hundreds of Japanese fighter court on charges of spying. planes descended on the base, where ● The order by the Hague-based they managed to destroy or damage International Court of Justice (ICJ) came nearly 20 American naval vessels, a day after India approached it against including eight battleships, and over 300 the death sentence handed down to airplanes. Jadhav by Pakistan’s Field General Court ● More than 2,400 Americans died in the Martial. attack, including civilians, and another ● India, in its appeal to the ICJ, accused 1,000 people were wounded. The day Pakistan of “egregious” violations of the after the assault, President Franklin D. Vienna Convention on Consular Relations Roosevelt asked Congress to declare war and asserted that Jadhav was kidnapped on Japan. from Iran where he was involved in 63. Ans. B. business activities after retiring from the ● The Hubble Space Telescope is a large Indian Navy but Pakistan claimed to have telescope in space. It was launched into orbit by space shuttle Discovery on April

34 www.gradeup.co

24, 1990. Hubble orbits about 547 titles, which includes eleven Superseries kilometers (340 miles) above Earth titles. ● Edwin Hubble, for whom the Hubble 68. Ans. A. Telescope is named, used the largest ● An Indian-origin British professor of telescope of his day in the 1920s at the chemistry and DNA expert at Cambridge Mt. Wilson Observatory near Pasadena, University Shankar Balasubramanian Calif., to discover galaxies beyond our received Knighthood from Queen own. Elizabeth II for their contributions. ● Hubble, the observatory, is the first ● He was recognised for his work as a co- major optical telescope to be placed in inventor of Next Generation DNA space, the ultimate mountaintop. sequencing (also known as Solexa 64. Ans. D. sequencing), described as the most ● Norway has become the world's first transformational advance in biology and country to shutdown the Frequency medicine for decades. Modulation (FM) radio broadcasting About Knighthood network and started its' switching to ● It is one of the highest honours Digital Audio Broadcasting (DAB) that bestowed upon an individual in the UK offers a wider range of broadcasting and it doesn’t carry any military options and greater sound quality. obligations to the sovereign. Note: ● Persons receiving it are entitled with ● Norway capital is Oslo. title ‘Sir’ before their name. The ● The currency of Norway is krone. knighthoods are conferred by the Queen 65. Ans. D. or a member of the Royal Family acting ● Emmanuel Macron took oath as on her behalf in Britain. Queen usually France’s next president on May 14 2017. presents insignia at the ceremony. ● Mr. Macron appeared beside Mr. 69. Ans. B. Hollande at a ceremony at the Arc de ● Air India flew the “world’s longest” all- Triomphe to observe the 72nd women-operated flight from Delhi to San anniversary of the end of World War II in Francisco. Europe ● The flight, which travelled a distance of 66. Ans. C. around 14,500 kilometre in close to 17 ● Till the end of 2016, the total number hours, was operated as part of of UNESCO's World Heritage Sites in International Women’s Day celebrations. India is 35. ● The non-stop Delhi-San Francisco flight ● In 2019, the Walled City of Jaipur took off from the national capital on (Rajasthan) known for its iconic March 6, 2016. architectural legacy and vibrant culture, 70. Ans. C. on made its entry into the UNESCO World In long-pending reforms that came into Heritage Site list. effect, emerging and developing ● The announcement was made after the economies gained more influence in the 43rd Session of the UNESCO World governance architecture of the Heritage Committee, at Baku International Monetary Fund (IMF). (Azerbaijan). ● India’s voting rights at the International ● With this, India now has 38 World Monetary Fund (IMF) increase to 2.6 per Heritage Sites (30 cultural properties, 7 cent from the current 2.3 per cent, natural properties and one mixed site). and China’s, to six per cent from 3.8, as 67. Ans. C. per the new division. ● Among the following, Saina Nehwal has ● Russia and Brazil are the other two won the maximum number of titles. countries that gain from the reforms. ● Saina Nehwal is an Indian professional ● More than six per cent of the quota badminton singles player. A former world shares will shift to emerging and no. 1, she has won over 24 international developing countries from the U.S. and European countries.

35 www.gradeup.co

● The combined quotas — or the capital ● The name "Juno" comes from stories countries contribute — doubles to about told by the Romans long ago. In the $659 billion from about $329 billion. stories, Juno was the wife of Jupiter. The emerging economies gained more Jupiter hid behind clouds so no one could influence in the governance architecture see him causing trouble. But Juno could of the International Monetary Fund (IMF). see through the clouds. ● The reforms, were agreed upon by the ● And the Juno spacecraft can see 188 members of the IMF in 2010, in the through the clouds of the planet Jupiter. aftermath of the global financial Juno is helping scientists study Jupiter. meltdown. 75. Ans. D. ● More than six per cent of the quota ● Belgium became the first country to shares will shift to emerging and allow euthanasia for terminally ill children developing countries from the U.S. and of any age. European countries. ● A terminally ill minor has become the Note: first child to be euthanized in Belgium ● IMF headquarters is in Washington, since age restrictions were lifted in the D.C, US. country two years ago, according to 71. Ans. A. several sources. ● Myanmar's new president has been ● The child, who was suffering from an sworn in, the first elected civilian leader incurable disease, had asked for in more than 50 years. euthanasia. ● In March 2016, Myanmar's parliament ● In 2014, the bill extended the "right to has elected Htin Kyaw as the country's die" to those under the age of 18. But next president, the first civilian leader there were additional strict conditions, after more than 50 years of military rule. including that the child was judged to be ● Htin Kyaw is a close ally of Aung San able to understand what euthanasia Suu Kyi, whose National League for means. Democracy (NLD) party swept to victory Note: in historic elections. ● The capital of Belgium is . 72. Ans. D. ● Euro is the currency of Belgium. ● In June 2016, the world's longest and 76. Ans. B. deepest rail tunnel has officially opened in ● Pakistan approves Hindu Marriage Switzerland, after almost two decades of Bill. construction work. ● Pakistan’s lower house of Parliament ● The 57km (35-mile) twin-bore has passed a landmark bill giving its small Gotthard base tunnel will provide a high- Hindu minority the right to register speed rail link under the Swiss Alps marriages, the last major hurdle on the between northern and southern Europe. way to enacting a law aimed at protecting 73. Ans. C. women’s rights. ● Solar Impulse is a Swiss long-range ● Activists say that Hindu women have experimental solar-powered aircraft been disproportionately targeted for project, and also the name of the abduction, forced conversions and rape project's two operational aircraft. because their marriages were never ● The Solar Impulse project's goals were officially recognised and therefore not to make the first circumnavigation of the provable in court. Earth by a piloted fixed-wing aircraft 77. Ans. C. using only solar power and to bring ● The 2016 Nobel Peace Prize was attention to clean technologies. awarded to the President of Colombia 74. Ans. A. Juan Manuel Santos "for his resolute ● Juno is a NASA spacecraft. It is efforts to bring the country's more than exploring the planet Jupiter. Juno 50-year-long civil war to an end, a war launched from Earth in 201A. It reached that has cost the lives of at least 220,000 Jupiter in 2016. That was a five-year trip!

36 www.gradeup.co

Colombians and displaced close to six ● Ashwin achieved the milestone in his million people. 37th Test match, bettering Waqar and 78. Ans. B. Lillee, who both took 38 Tests. ● Immediately before Antonio Guterres 81. Ans. A. was appointed the U.N Secretary General ● The Happiness Index Department or a in October 2016, he was United Nations Wing has been established in the states High Commissioner for Refugees. of Madhya Pradesh and Andhra ● On 13 October 2016 António Guterres Pradesh. was appointed by the General Assembly ● In April 2017, Andhra Pradesh became as the ninth Secretary-General of the the second state in the country, after United Nations, for a five-year term from Madhya Pradesh, to set up a department 1 January 2017. dedicated to happiness. 79. Ans. C. ● In the World Happiness Report released ● With the development of Terahertz in March 2018, India was ranked 133rd (THs) transmitter, it is expected to be out of 156 countries. faster than 5G mobile networks by ten ● According to World Happiness Report times. 2019 India's ranking went down to 140. ● Scientists have developed a next ● Finland, for the second consecutive generation system which can transmit year, has topped this list. It is followed by digital data over 10 times faster than 5G Denmark, Norway, Iceland, Netherlands, mobile networks, an advance that will Switzerland, Sweden, , pave the way for faster downloads and Canada and Austria. improve in-flight network connection 82. Ans. C. speeds. ● Tamil Nadu is the country’s top ● Researchers from Hiroshima University destination for domestic tourists for the and National Institute of Information and third consecutive year. Communications Technology in Japan ● More than 344 million tourists from have announced the development of a within the country visited the state in terahertz (THz) transmitter capable of 2016 transmitting digital data at a rate ● Domestic tourist arrivals in Tamil Nadu exceeding 100 gigabits per second over a in 2016 registered a 3% increase over the single channel using the 300-gigahertz previous year. band. 83. Ans. D. 80. Ans. D. ● COIN, a software programme ● Ravichandran Ashwin he became the developed by J. P. Morgan supports fastest bowler to reach 250 wickets. The Interpreting commercial documents. Indian spinner surpassed Australian ● At JPMorgan, a learning machine is legend Dennis Lillee's record by taking parsing financial deals that once kept just 45 matches to reach the landmark -- legal teams busy for thousands of hours. three matches quicker than the former ● The program, called COIN, for Contract Aussie pacer. Intelligence, does the mind-numbing job ● Ashwin had a record-breaking last year of interpreting commercial-loan for India as he picked up 72 wickets in 12 agreements that, until the project went Test matches to finish as the highest online in June, consumed 360,000 hours wicket-taker in the world for the second of lawyers’ time annually. The software consecutive year. reviews documents in seconds, is less ● Ashwin had earlier beaten Pakistan error-prone. great Waqar Younis and Lillee to become 84. Ans. A. the second fastest bowler to take 200 ● NASA rediscovered India's lunar Test wickets during the first Test against spacecraft that was lost in the space New Zealand in September last year. during the past eight years known as Chandrayan – I.

37 www.gradeup.co

● Eight years after ISRO lost hope and around 400,000 asylum claims created by considered it lost, India’s first lunar the unprecedented number of refugees spacecraft, Chandrayaan-1, has been re- who arrived in Germany in 2015. discovered by NASA’s radars. 87. Ans. B. ● In India’s first lunar probe, ISRO had ● In 2017, Sri Lanka’s Right to launched Chandrayaan-1 launched in Information (RTI) Act comes into effect October 2008 and was operational until bringing with it a promise of open 2009. government, citizens’ active participation ● After it was launched on 22 October in governance, and accountability to the 2008, on 8 November 2008, the Moon people of the country. Impact Probe separated from the ● The Ministry of Mass Media and Chandrayaan orbiter and struck the Parliamentary Reform said that it has South Pole which made India fourth been laying the groundwork of RTI, with country to place its flag on the moon. trainings for public officials and About Chandrayaan II appointments of the key information ● Chandrayaan-2 is India's second lunar officers and designated officers. exploration mission after Chandrayaan-A. ● The ministry said they are working Developed by the Indian Space Research closely with the RTI Commission, which is Organisation, the mission was launched vested with wide powers under the Act, from the second launch pad at Satish such as to hear appeals, institute action Dhawan Space Centre on 22 July 2019 at against alleged offenders, and prescribe 2.43 PM IST to the Moon by a guidelines on record management and Geosynchronous Satellite Launch Vehicle proactive disclosure. Mark III. 88. Ans. B. ● Chandrayaan-2 will be the first-ever ● As on 31st January 2016, the highest mission to land a rover near the lunar number of law colleges were present in south pole. Uttar Pradesh. ● It includes a lunar orbiter, lander and 89. Ans. D. rover, all developed by India. ● The first statue of a woman in 85. Ans. C. Parliament Square in England is that of ● In a first, an unusual home has been Millicent Fawcett. 3D-printed by a Russian startup ● The first statue of a woman to reside in company in less that a day using a the famous square opposite Westminster portable machine. Palace. ● A mobile 3D printer created the ● The Fawcett statue joins 11 of men in building’s concrete walls and partitions as the square, including Winston Churchill, a fully connected structure, rather than Nelson Mandela and Mahatma Gandhi. printing the building in panels at an off- Note: site facility as is usually done. ● Fawcett was a leading figure in the ● The portable machine was then suffragist movement. Born in 1847 in removed from the building, and the home coastal town of Aldeburgh, Fawcett was was completed by adding the roof and interested in women’s suffrage, even as a windows, and finishing the interior. The teenager. cozy, 37-square-meters home with an ● When she was 19, she organized unusual, curved shape has all of the signatures for a petition calling for votes standard features of a traditionally built for women (she was too young to sign it house. herself). 86. Ans. C. ● She became well-known as an advocate ● Germany offers asylum seekers up to for peaceful protests and in 1897 became €1,200 each to voluntarily return to their president of the National Union of home countries. Women’s Suffrage Societies. She also ● The programme was launched as civil campaigned for equal access for higher servants continue to battle a backlog of education.

38 www.gradeup.co

● She died in 1929, a year after voting said contract entered into amounts to rights were extended to all women older fraud. than 21. 95. Ans. C. 90. Ans. B. An agreement to get a person ● India emerged as Twelfth largest employment in the public service i.e. holder of the U.S Government Securities Government is absolutely opposed to at the end of 2016. public policy. The principle states that ● India became the 12th largest holder of agreement, opposed to public policy, is US government securities at the end of void. Hence the said agreement is void. 2016, with exposure worth USD 118.2 96. Ans. B. billion. The principle states that causing bodily ● With holdings worth USD 1.09 trillion, injury to another person by use of Japan remained the largest holder of physical force amounts to Assault. The these securities followed by neighbouring fact mentions that although Rustum China with exposure to the tune of USD raised his fist in anger he did not hit him, 1.06 trillion. thus as per the principle Rustumhas not ● Latest data available with the US committed assault. Treasury Department showed that India's 97. Ans. A. holding stood at USD 118.2 billion in The facts clearly states that A has sold his December last, slightly lower than USD car to B. Thereafter B requested A to keep 118.7 billion seen in November. the said car. Although the car is in the ● At the end of December 2015, India's possession of A, that by no stretch of exposure was to the tune of USD 116.8 imagination means that A has not sold billion. the car. 91. Ans. A. 98. Ans. D. A, offered to sell his motorcycle for Rs. X made a promise to Y to repair his car 25,000, whereas B wanted to buy the engine. But Y met with an accident due to same for Rs. 24,000. B never accepted bursting of the tyre, there was no the offer made by A. Hence, B cannot problem with the car engine thus Y is not claim any damages. liable to pay compensation. 92. Ans. B. X never promised to Y to repair his tyre. The fact clearly mentions that Mr. Martin, Had X made a promise to Y to repair his found the Key of Audi A3 Car and then car trye, X would have been liable to pay came to know about the Company's compensation. advertisement. As per the principle, 99. Ans. A. promise to do something in return for a Admittedly the P had given his consent to benefit is enforceable by law, thus Mr. undergo a a surgical operation for Martin can claim the car. removal of appendicitis, the surgeon 93. Ans. B. while doing surgery also removed the gall The doctor offered to treat the patient for bladder which was never consented by P. a consideration of Ten Lakh and the hence as per the principle P can claim patient accepted the offer. the doctor did compensation. not use his dominent position to influence 100. Ans. A. the will of the patient, hence the contract The condition imposed by Sunder is enforceable by the doctor. “provided the house was put to thorough 94. Ans. A. repairs and the living rooms were Zameer by deliberately concealing the decorated according to contemporary fact that he had undergone a medical style” is ambiguous and not capable of treatment for a serious ailment has being made certain/ definite. Hence the committed fraud on the insurance agreement is void. company, thereby inducing the insurance 101. Ans. D. company to enter into a contract. The The principle states that takes away a girl who is less than eighteen years of age out

39 www.gradeup.co

of the custody of parents of such minor 108. Ans. B. without the consent of such parents, is B has not committed the act fraudulently said to commit no offence. Hence A has since he never intended to defraud A. had not committed no offence. B withdrawn some money from A’s 102. Ans. D. account it would have been an offence The principle states that “When a person under the principle. The intention to interferes with peaceful possession of defraud is missing. another person without the permission, 109. Ans. D. commits trespass” P1 had displayed a Leaving a cotton swab inside the notice that it is not a thoroughfare even abdomen is not expected out of a doctor, then T1 entered into P1’s land thus have it is simply a failure on the part of the committed trespass. doctor to take proper care of his patient, 103. Ans. B. it amounts to negligence. Hence, the The principle states “whoever voluntarily doctor is liable for negligence. has carnal intercourse against the order 110. Ans. B. of nature” against a man or women Keeping smuggled goods is an immoral commits rape. Admittedly a man was and illegal activity. Thus as per the found engaged in carnal intercourse with principle the contract between P and R is an animal and not against a man hence invalid. P is not bound to keep the the person cannot be held liable for rape. smuggled goods since the contract is for 104. Ans. D. an immoral or illegal purpose. Although A is under the influence of 111. Ans. A. madness that does not mean the other The fact that M asked S to do a particular person cannot use private defense to work (submit a report at the Government protect his life. Hence B can use private Labour Office) and that while doing that defense to save his life. work S was hit by the Truck and was 105. Ans. A. thereafter admitted to a hospital. As per Defamation means lowering the the principle the employer will be liable as reputation of one person in the eyes of the accident took place during the course another person. An offence of defamation of employment. consists of three-person i.e. 1) The 112. Ans. C. person making the defamatory The facts specifies the difference in statement, 2) the person to whom the opinion between the workmen and the defamatory statement is made and 3) in employer with regards to the declaration whose eye the reputation is lowered. of half a day work on lunar eclipse which Hence, A has committed defamation. was refused by the employer. Hence 106. Ans. C. there is a dispute with regards to the The fact states “A was unwilling to employment hence the matter amounts execute the deed, but he was forcefully to industrial dispute. restrained by P and his body guards in P's 113. Ans. D. office and made A sign the gift deed” this Omission on the part of the company to means he was forced to execute the deed inform the public about the dismissal of thus as per the principle A executed the Rahul has led the customer to be deed under compulsion hence A was right defrauded by Rahul hence the company in withdrawing from the contract which shall be held liable for the fraudulent act he entered into under force. OF Rahul. 107. Ans. D. 114. Ans. C. The principle talks about “right to a fair Although R offered to sell his old car for a hearing” in the instant fact X was never price of Rs. Three lakhs, but S offered to given the opportunity to put forth his part buy the same for Rs. 2 lakhs 50 thousand of the story. Thus by not giving an and R agreed to sell it for Rs. 2 lakhs 50 opportunity to X, the employer violated thousand, means that the offer made by the principle of natural justice. S to buy the car for Rs. 2 lakhs 50

40 www.gradeup.co

thousand was accepted by R. Hence it’s a The principle states that one can use valid contract. necessary force against wrong-doer for 115. Ans. B. the purpose of protecting one's own body Arbitration, a form of alternative dispute and property. In the instant factual resolution (ADR), which aims to resolve situation, X shot at the dog when there disputes outside the courts. Thus, the was no eminent threat/ danger. Hence agreement (arbitration clause) is valid in the use of such force is not protected by the eyes of law. law. 116. Ans. D. 121. Ans. D. The principle states that no suit or other As per the principle promissory note duly legal proceeding shall be maintainable in executed in favour of a minor is not void, any civil court with respect to any officer and nothing in the Contract Act prevents or member of trade unions in India with him from making the other party bound regards to any act done by him in to the minor, hence the minor can accept contemplation or in furtherance of a trade the benefit arising out of the contract. dispute. 122. Ans. B. Admittedly Soloman took the loan for his The fact that P was requesting the film personal purpose i.e. for the higher star for the actual expenditure incurred education of his daughter, hence the by him is legally sustainable. Bank can file a suit for recovery of the 123. Ans. B. loan. It is not a trade dispute. Note – only when there is a violation of a 117. Ans. C. right, compensation can be claimed. Sanju, Dilbag and Sushil decided to Since T has not violated any of ABCCs commit burglary, it was only Sanju who legal right, T is not liable to compensate committed rape hence it is only Sanju ABCC. who shall be held liable for rape and the 124. Ans. D. others ie. Dilbag and Sushil would be held David is guilty of forgery since he was liable for burglary. trying to falsifies something with the 118. Ans. D. intent to deceive another. When a person dishonestly converts 125. Ans. D. another movable property to his own the John has made additions to make the said act is known as misappropriation. X contents clearer, thus there was no was responsible to take care of the intention to deceive another. Thus the valuables and cash of A. X addition which was intended to make misappropriates the said valuables and things clearer did not change the original cash and uses the same to drink liquor. content. Hence the instant fact does not Hence X is liable for misappropriation and indicate a case of forgery. embezzlement. 126. Ans. B. 119. Ans. C. Bona vacantia is a legal term for the Fundamental Rights given under part III situation in which property/ goods are left of the Constitution is only guaranteed by without any clear owner the State and enforceable against the 127. Ans. C. State. It is the duty of the State to protect Caveat Venditor is a Latin term which the Fundamental Rights of the citizens. In means let the seller beware. The person case a Fundamental Right which is selling goods is accountable for providing guaranteed by the State is violated by an information about the goods to the seller. action of the State then the individual has 128. Ans. A. a right to approach the Court under Writ Faux pas means a significant or jurisdiction. In case a fundamental right embarrassing error or mistake. is violated by an action of a private 129. Ans. D. individual there is no remedy available. In pari delicto refers to a situation when 120. Ans. B. two persons are equally at fault. 130. Ans. A.

41 www.gradeup.co

lex loci refers to "the law of the place" - law of the country in which a transaction is performed 131. Ans. B. Malus animus means the intention to do harm; the intention to commit an illegal or immoral act. 132. Ans. C. Per incuriam refers to a judgment of a court which has been decided without reference to a statutory provision or earlier judgment which would have been relevant. 133. Ans. D. Hence, A is the Singer. Pari-passu is a Latin phrase meaning 142. Ans. C. "equal footing" 1) Three of them - A, C and Doctor prefer 134. Ans. A. rice to chapatti and two of them - B and Defendant's plea stating that he or she the Painter prefer chapatti to rice. has already been tried for and convicted 2) The Teacher, D and A are friends to of the same offense. one another but two of these prefer 135. Ans. C. chapatti to rice. lis pendens refers to 'a pending legal action'. 136. Ans. C. Sine die refers to a situation where a matter is adjourned without any future date being designated. 137. Ans. B. 'Punctum Temporis' refers to a point of time. 138. Ans. C. Turpis arbiter refers to a corrupt judge. 139. Ans. C. Jus Gentium refers to the concept of international law. 140. Ans. D. Hence, B is the Teacher. Animus posssidendi refers to the 143. Ans. B. intention to possess. Final arrangement is given below: 141. Ans. A. 1) Three of them - A, C and Doctor prefer rice to chapatti and two of them - B and the Painter prefer chapatti to rice. 2) The Teacher, D and A are friends to one another but two of these prefer chapatti to rice.

Hence, C is the Dancer. 144. Ans. C.

42 www.gradeup.co

1) R, next to U gets a South-facing flat Exact is redundant before estimate. and T gets a North-facing flat. There is not word like exact estimate. 2) Q gets a North-facing flat and is not Similarly, only does not makes any sense next to S. S and U get diagonally opposite before choice. Also, open secret does not flats. make any sense. While, ‘clearly visible’ is word exists and makes sense as an adjective before visible. 150. Ans. B. Clearly, U, R and P get South-facing flats. Cruise is odd from the given options - 145. Ans. A. Expedition - a journey undertaken by a 1) R, next to U gets a South-facing flat group of people with a particular purpose and T gets a North-facing flat. Cruise - sail about in an area without a 2) Q gets a North-facing flat and is not precise destination, especially for next to S. S and U get diagonally opposite pleasure. flats. Crusade – a vigorous campaign for political, social, or religious change. Campaign - work in an organized and active way towards a particular goal,. Clearly, T’s flat is between Q and S. 151. Ans. B. 146. Ans. D. Family tree:- 1) R, next to U gets a South-facing flat and T gets a North-facing flat. 2) Q gets a North-facing flat and is not next to S. S and U get diagonally opposite flats.

If the flats of T and P is interchanged, then clearly, R’s flat will be next to that of U. If use assumes the grandfather to be 147. Ans. D. parental then grandfather only son will be 1) Statement 1 says that there are some father. Thus prakesh’s grandfather only men are naturally good. i.e.- Which is son’s daughter will be prakash’s sister. partially correct Hence, prakash will be the brother or 2) Statement 2 says against the sister of the girl. Now, according to the statement. options given questions Y we conceder So, this inappropriate prakash as male. Then they will be the 3) Statement 3 speaks against the brothers of the girl. statement. 152. Ans. D. 4) Statement 4 says all men are naturally good which is correct and best option twice negation is same as position & definite statement. 148. Ans. A. 1) Statement 1 is the correct and best If we consider Ravi as male then Ravi’s explanation of the alone daughter’s father will be Ravi himself . Next statement Speaks against the Now, Ramesh is the son of his daughter’s statement father thus, Ramesh is Ravi’s son. Only bare A is equivalent to all A and B. 153. Ans. D. Hen+ce 1 is correct Consider the diagram 149. Ans. C.

43 www.gradeup.co

be either mother or aunt of Ram, but exact relation can't be determine. 159. Ans. D.

Hence, the bus facing towards west. 154. Ans. D.

Ravi’s younger brother’s grandmother will be Ravi’s grandmother Ravi’s grandmothers only son will be Ravi’s

father and Ravi’s grandmother’s only son’s uncle will be Ravi’s mother. Thus, Ravi and the boy in photo are siblings i.e. brothers 160. Ans. A.

Since we don’t know crossing is in which direction, so, multiple cases are possible and hence answer is cannot be determined. 155. Ans. A.

161. Ans. D.

156. Ans. C. Here ”Seldom” is negative thus meanswer that these books are not read. Hence (A) is correct. 157. Ans. B.

Consider the diagram 162. Ans. A. Clearly, Vaishnavi prefers History least. As after going through options, Economics is preferred over Maths. Maths is preferred over History and also Social is preferred over Maths. 163. Ans. A.

Hence, the most appropriate conclusion is No crows are parrots. 158. Ans. D. Since, Girl's mother's brother is only son of Ram's mother's father; it means either

Ram and Girl are siblings or cousins of 164. Ans. C. each other. Therefore, girl's mother will

44 www.gradeup.co

Crumbs are pieces of bread. Water is not (Note: Leap year had nothing to do with piece of vessel. Inch is not piece of unit. the question) But splinters are pieces of wood. 173. Ans. D. 165. Ans. D. A + 3 = D + 3 = G + 3 = J Apostate- A person who renounces X – 3 = U – 3 = R – 3 = O religions. Thus, missing term = JO. 166. Ans. C. 174. Ans. B. Misologist: Hater of Knowledge and As a mare is a female adult horse leaning. similarly a sow is a female adult boar. 167. Ans. B. 175. Ans. B. The occurrence of the same letter or 9:72:8:? sound at the beginning of adjacent closely 64 connected words. 9×8=72 168. Ans. B. 8×7=64 If we observe this is a simple 176. Ans. B. rearrangement of words in which the first The code for MILITARY is given as term of the original is replaced by the first 12324567. It’s the number pattern term of the replaced word. For example, between 1 to 7, straightforward usage of A of “ABANDON” becomes “a” of numbers for the word MILITARY only 2 'aramoim' and D of “ABANDON” becomes interprets because of the multiple usage “o” of 'aramoim'. By this logic, original of letter 'I'. Therefore, for the Word TAIL word of 'bituo' will be “SOLID”. can be taken from MILITARY for TA we 169. Ans. B. can take 45 and for IL we can take 23. Clearly, 11 times, the hands of a clock are So, for TAIL we can code as 4523. at right angles from 4 pm to 10 pm. 177. Ans. A. 170. Ans. B. The shop owner had 12 dozen and gave The grand-mother of my younger brother out 10 chocolates. A dozen equates 12 meanswer Ravi’s grand-mother and the chocolate. So, 12 dozens =144, when 10 only son meanswer Ravi’s father. Wife of is subtracted it becomes 134. The shop Ravi’s father is Ravi’s mother and hence owner added 24 (2 more dozen) which her son is the brother of Ravi. makes the number 158 and he divided it 171. Ans. A. into 2 equal parts. 158/2 =79. South east 178. Ans. B. 40% workers are females. This is equal to 800. So, 60% will be equal to (800 x 60)/40 = 1200. 179. Ans. B. Now, finding odd days March (4 days left) = 4 odd days; April (full month) = 2 odd days, May (full month) = 3 odd days; June (till 27) = 6 odd days Now, total odd days = 4 + 2 + 3 + 6 =

172. Ans. A. 15 i.e. 1 odd day Ms. X birthday = 4th October i.e. 27th June 2011 is 1 day after Sunday Ms. Y birthday = 28th September i.e. on Monday. Given, 15 August was on Sunday 180. Ans. C. Now, calculating odd days Aug (16 days The average human is made up of 50- left) = 2 odd days, Sept (till 28th) = 0 60% of water. odd day 181. Ans. D. i.e. 2 odd days. i.e. Ms. Y was born 2 days after Sunday Women = = Tuesday

45 www.gradeup.co

Married women = of = 2( ) = = = Women, who have children = × =

Men =

Married men = of = ⇒ = Given A left after 3 days & B & C Men, who have children = of = × completed the remaining work. Let x be the number of days taken by B & C to = finish the remaining work. So, part of workers without children = 1 - part of workers with children = 1 – ( 3( ) + x ( ) = 1 + = 1 – = 182. Ans. B. X + Y + Z = 75 × 3 = 225 kg 3( ) + x ( ) = 1 After A joins the group, ⇒ 4x = 19 X + Y + Z + A = 75 × 3 = 80 × 4 = 320 kg ⇒ A = (X + Y + Z + A) – (X + Y + Z) = ⇒ x = = 4 days 320 – 225 = 95 kg 184. Ans. C. ⇒ B = A + 5 = 95 + 5 = 100 kg Given, train A takes 1.5 hours (4:30 pm - Y + Z + A + B = 85 × 4 = 340 kg 3:00 pm) to go from A to B and train B Y + Z = 340 – (95 + 100) = 145 kg takes 1 hour (4:00 pm - 3:00 pm) to go X = 225 – (Y – Z) (Given above, X + Y + from B to A Z = 225) So, let distance AB = 3 km, then ⇒ X = 225 – 145 = 80 kgs Speed of train from A is 2 km/hr and that 183. Ans. B. of B is 3 km/hr Let the number of days A, B & C take to Now time taken by trains to meet each complete the work individually be a, b & c. other will be = 0.6 hours = 0.6 x 60 Given, A and B complete the work in 12 minutes = 36 minutes Hence, the required time = 3:00 pm + 36 minutes = 3:36 pm. days, + = 185. Ans. C. Given, B and C complete the work in 8 Let the required fraction be x, then Final fraction of milk = initial fraction of milk × (1 – fraction of mixture taken out)n days, + = Where ‘n’ is the number of times process Given, C and A complete the work in 16 is done

Thus, = × (1 – x)1 days, = Adding all the 3 equations, ⇒ 1 – x =

⇒ x = 186. Ans. D.

46 www.gradeup.co

Let tap B be turned off after X minutes, ⇒ 4y – 5x = 20 then ⇒ 4× – 5x = 20 (y = ) ( + ) = 1 ⇒ 300 – 5x2 = 20x ⇒ x2 + 4x – 60 = 0 ⇒ = 1 – = 1 – = ⇒ (x – 6)(x + 10) = 0 ⇒ x = 6, -10 Length cannot be negative, so the length ⇒ x = = 9 of the cloth is 6 meters 187. Ans. A. 190. Ans. A. Let the velocity of boat in still water be b Banker’s discount (BD) for 1.5 years = k.ms/h and velocity of current is c k.ms/h Rs. 600 Given: Banker’s discount (BD) for 3 years = Rs. 1200 = True discount (TD) for 3 years = Rs. 750 ⇒ 5(b – c) = 3(b + c) ⇒ b = 4c Sum = Now,

= = 12 = Rs. 2000 Thus, Rs. 1200 is the S.I. on Rs. 2000 for ⇒ = 12 3 years

⇒ = 12 So, rate = = 20% 191. Ans. B. ⇒ = 12 Let the number of notes of Rs. 2, Rs. 5 ⇒ 9 + 15 = 12c and Rs. 10 be 3P, 5P and 8P respectively, then ⇒ c = = 12 (2 × 3P) + (5 × 5P) + (10 × 8P) = 6P + ∴ Velocity of current is c k.ms/h = 12 25P + 80P = Rs.4662. k.ms/h ⇒ 111P = 4662 188. Ans. C. ⇒ P = 42 Let the trader buy 100 kg rice in Rs. 100 Hence, answer = 5P = 5 × 42 = 210. and he sells 90 kg (100 kg – 10% of 100 192. Ans. C. kg) in Rs. 120 The clock loses 10 minutes per hour, then the clock shows only 50 minutes for 60 minutes in 1 hour. So, the total gain earned by him = ( × Then after 1 hour, clock will show time 12: 50, 100 – 100) = 33 %. Alternate Method: ∴ The angle = 30h – = (30 x 12 –

Gain% = × 100 = 33 )º = (360 – 275)º = 85º 193. Ans. A. % Let the rate of interest be R% 189. Ans. D. For 2 years the difference between simple Let the length of the cloth be x meter and the cost of 1 meter be Rs. Y and compound interest = A.T.Q, xy = (x + 4)(y – 5) = 75 ⇒ xy = xy + 4y – 5x – 20 ⇒ 98 = ⇒ R2 = 196

47 www.gradeup.co

⇒ R = 14 x – 3y = -20 ------(i) 194. Ans. B. Also, (x + 10) = 2(y + 10) x – 2y = 10 ------(ii) On solving equations (i) and (ii), we get Y = 30 and x = 70 Hence, the required answer = 70 : 30 = 7 : 3 200. Ans. C. Let his father gave him Rs. X

The ratio in which two metals are mixed = 8 : 2 = 4: 1. He spent in buying books and 195. Ans. D. entertaining his friends, then remaining A, B and C together completes the work in 4 days, one day work of all three is amount = Rs. He deposited half of remaining amount in + + = his savings i.e. , then he left with Rs. ⇒ + + = Now, – 5 = 20 ⇒ = - X = 25 × 4 = 100

=

Efficiency of A, B and C = : : = 5 : 4 : 1

C’s share = × 720 = Rs. 72 196. Ans. B. Let required number of days be x, then A.T.Q, 14(2M + 7B) = 11(3M + 8B) ⇒ M = 2B x (8M + 6B) = 3 × 14(2M + 7B) ⇒ x (16B + 6B) = 42(4B + 7B) ⇒ 22Bx = 42 × 11B ⇒ x = 21 197. Ans. D. Let the distance be D km A.T.Q,

= 5 ⇒ D = 5 × 3 = 15. 198. Ans. A. Probability of getting both the ball of same color = × + × = 199. Ans. D. Let the father’s present age be x years and the son’s present age b y years A.T.Q, (x – 10) = 3(y – 10)

48 www.gradeup.co

49